Pediatrics, Glaucoma, ADHD, Lupus , INFLUENZA,

Réussis tes devoirs et examens dès maintenant avec Quizwiz!

Colchicine should NOT be used with macrolides (especially clarithromycin) due to death related __________________.

agranulocytosis

A flare is a measurable increase in disease activity in one or more organ systems involving new or [worsening] clinical signs and symptoms and/or laboratory measurements. It must be considered clinically significant by the assessor and usually there would be what?

at least a consideration of a change or an increase in treatment

With the goals of control symptoms, prevent complications, decreased work/school absence, prevent spread. Influenza treatment is Most effective within 48 hrs of illness onset -The sooner, the better (more effective) -Ideally, with diagnosis What medications are used for treatment?

-Amantadine and rimantidine - not Rec. anymore -Baloxavir marboxil (XofluzaTM) -Oseltamivir (Tamiflu®), zanamavir (Relenza®), peramivir (Rapivab®)

Inattention:

-Failing to show close attention to details -Difficulty organizing tasks and activities -Forgetful

Medication Treatment Options Increase aqueous outflow:

-Prostaglandin analogs -- 1st Line -Alpha2 agonists -Cholinergic agonists -Rho kinase inhibitor

BSA rough estimate:

-area NOT used in the PASI score is palm size (1% BSA)

Metabolic syndrome consists of 3 or more of the following:

-waistline: ≥40 inches (men) or ≥35 inches (women) -BP: ≥130/85 mmHg or taking BP meds -Triglyceride levels >150mg/dl -Fasting blood glucose >100 mg/dl or taking glucose lowering meds -HDL <40 mg/dl (men) or 50 mg/dl (women)

Colchicine is 1st line for anti-inflammatory prophylaxis during initiation of prophylactic therapy. 1. What is the ideal length of treatment? 2. What is the practical length of treatment?

1. 6 months (no increased risk of flare after discontinuation) 2. Minimum 8 weeks (still increased risk of flare)

Match the following specific AE to the correct medication: 1. Allergic reaction (30%), Edema, pain, itching, foreign body sensation; [less lipophilic, fewer systemic SEs] 2. Allergic reaction (8%); more α2 selective, fewer ocular SEs

1. Apraclonidine 2. Brimonidine

Normal neuron function flow of information:

1. Dendrites: collect electrical signals 2. Cell body: Integrates incoming signals and generates outgoing signal to axon 3. Axon: Passes electrical signals to dendrites of another cell or to an effector cell

Treatment Goals & Objectives

1. Goal = Remission 2. Involve patient in treatment decisions - understand their goals and expectations 3. Tailor therapy to disease severity and extent of organ impairment

Pathophysiology of rheumatoid arthritis involves inflammation of the synovial tissue lining the joint. What happens during this process?

1. Immune system attacks the synovial and other connective tissues 2. Chronic inflammation leads to proliferation of the tissue, results in a pannus (abnormal layer tissue) Eventually the pannus invades the cartilage then the bone, ultimately causing destruction of the joint

Match the following Bacterial Vaccines to their correct formulations: 1. Endotoxin 2. Conjugated Polysaccharide

1. Inactivated; Weakened(live) 2. Polysaccharide

Methylphenidate Drug Interactions

1. MAOI: Selegiline, Tranylcypromine, isocarboxazid, phenelzine, linezolid, methylene blue 2. Antihypertensive drugs 3. Halogenated anesthetics: avoid day of surgery

Arrange the following vehicles from the MOST occlusive to the LEAST: -ointment, creams, lotions, solutions, gels, sprays

1. Ointment 2. Creams 3. Lotions 4. Solutions 5. Gels 6. Sprays

1. Desired therapeutic outcomes for the use of 1st line acute gout attack agents (NSAIDs, corticosteroids, colchicine) is? 2. When monitoring prophylaxis treatment of gout, check serum urate levels every 2-5 weeks during titration then every 6 months.

1. Resolution of pain 2. False

Dextroamphetamine Drug Interactions:

1. Serotonin Syndrome °CYP2D6 inhibitors - paroxetine, fluoxetine, quinidine, ritonavir °SSRI, SNRI, triptans, tricyclic antidepressants, fentanyl, lithium, tramadol, buspirone, St. John's Wort 2. Avoid use with antacids - inc. dextroamphetamine conc. 3. MAOI Inhibitors 4. Antihypertensives

Treatment for Chronic Airway Infection in CF:

28-day every other month cycles Newly acquired: Inhaled tobramycin Chronic: All Inhaled tobramycin, aztreonam, vancomycin

Interferon-β1b (Betaseron) you must use within _______ of mixing!

3 hrs

*KNOW* Do NOT use high potency corticosteroids for more than?

3 weeks apply twice daily, up to 4 times daily (may not result in better results)*

How do we monitor NSAIDs for safety?

A. Monitoring renal function & BP frequently B. BMP & CBC C. Monitoring liver function D. Labs and symptoms monitored periodically with frequency depending on underlying risk

Non-pharmacological therapy for OA includes:

A. Patient education B. Weight loss C. Exercise D. Muscle strengthening exercises E. Tai Chi F. Cane/Knee brace G. Orthosis- 1st carpometacarpal

Non-pharmacological interventions for psoriasis include:

A. Stress-reduction strategies B. Moisturizes (non-medicated) C. Oatmeal baths D. Use of sunscreens with SPF 30+ E. Minimize irritation F. Manage comorbidities such as dyslipidemia, obesity, CV disease, smoking, alcohol

Effects are seen fastest (within 1 week) with which of the following agent?

Adalimumab (Humira): 80 mg SQ first week, then 40 mg SQ the next week, then every 2 weeks thereafter

Carbachol (Carboptic, Isopto Carbachol) Direct cholinergic effects and More resistant to cholinesterase than pilocarpine. What is the dosing?

BID-TID

________________ is a cosmetic product for eyelash growth

Bimatoprost (Latisse) -Do not combine with glaucoma PG analog

What other med. has shown to be effective for Gait Difficulties and Spasticity?

Botulinum toxin (Botox) -Amount of toxin depends on muscle injected -Limited to smaller muscles due to amount of toxin that would be needed

Which muscle relaxant should be avoided?

Carisoprodol (Soma) - dependence, abuse, diversion; increase concentrations with poor CYP2C19 metabolizers

*QUESTION* Which one of the following regimens would be the BEST option to initiate in a treatment naïve patient with uric acid level of 8.1 mg/dL, and 3 attacks over the last year? A. Colchicine 1.2 mg now, followed by 0.6 mg 1 hour later B. Fenofibrate 48 mg daily + colchicine 0.6 mg daily x12 wks C. Febuxostat 80 mg daily + colchicine 0.6 mg daily x12 wks D. Allopurinol 100 mg daily + naproxen 220mg daily E. Lenisurad 200mg daily

D *IDEAL answer: Allopurinol 100 mg daily + colchicine 0.6 mg daily x12 wks*

*KNOW* Cyclosporine (an immunosuppressant) is used for moderate to severe psoriasis AND is only indicated for?

Exacerbations *NOT for chronic use due to side effects*

Topicals: Lidocaine Lidoderm patch (lidocaine 5%) - RX

FDA-approved for post herpetic neuralgia (PHN) ▪Very commonly used off-label for other types of neuropathic pain Apply 1-3 patches/day to affected area ▪Can be cut into smaller pieces ▪12 hours on, 12 hours off ▪Do not cover with heated covers

Vaccine Recommendations for MS:

Flu vaccine yearly: Including pts on DMTs varicella zoster immunization: Pts opting to take fingolimod who are varicella zoster antibody negative Intranasal (live, attenuated vaccine) is not recommended Live virus vaccines are also more likely to cause an increase in MS disease activity than inactivated virus vaccines

INFLUENZA VIRUS

Human influenza A: H1N1; H3N2 Human Influenza B: B-Victoria; B- Yamagata

What is the LEAST potent corticosteroid?

Hydrocortisone

What are dosing considerations regarding Abatacept (Orencia)?

IV infusion over 30 minutes, Weight based Dose IV at weeks 0, 2, and 4, then q 4 weeks thereafter >100 kg - 1000 mg 60 - 100 kg - 750 mg <60 kg - 500 mg

These agents can provide steroid-sparing effect and used when pts remain symptomatic after hydroxychloroquine (w/ or w/o steroids) or in those with organ-threatening disease at presentation:

Immunomodulating Therapies Non-biologic: cyclophosphamide, azathioprine, methotrexate, and mycophenolate -6 months or more: max therapeutic response -Use of more than one not recommended

Corticosteroids - methylprednisolone

Improve recovery by decreasing edema in area of demyelination

KNOW TOP 300 oral NSAIDs

KNOW DOSAGES TOO USE LOWEST EFFECTIVE DOSAGE

Which Beta Blockers may work QD?

Levobunolol and Timolol

The Gold standard diagnosis for Cystic fibrosis is what?

SWEAT CHLORIDE TEST Positive ≥ 60 mmol/L

T/F: Nalbuphine is used frequently in low doses to treat/prevent opioid-induced pruritis!

True

T/F: Oxycodone ER 60mg and 80mg tablets are NOT intended for opioid-naïve patients

True

Magnesium : Adverse Reactions

Weakness, malaise, hypotension, respiratory depression, arrhythmias Consider in patients with severe exacerbation who have no response after 1 hour of intensive conventional therapy

Miosis and hypotension are not what?

definitive for opioid overdose Naloxone IV/IM/SQ 0.4mg-2mg x1 -- CALL 911 May repeat after 2-3 min if needed up to 10mg total

What are Primary Symptoms of MS:

¨Visual complaints/ optic neuritis ¨Gait problems and falls / Ataxia ¨Paresthesias / Pain / Weakness / Fatigue ¨Spasticity ¨Speech difficulty / Psychological changes ¨Cognitive changes ¨Bowel/bladder dysfunction ¨Sexual dysfunction ¨Tremor

Risk factors for GI events include:

•Age >60 years •History of peptic ulcer or bleeding •Cardiovascular disease •Glucocorticosteroids •Antiplatelet drugs •Anticoagulants •NSAID dose

Treatment for Neuropsychiatric manifestations:

•Steroids and/or immunosuppressives for inflammatory causes •Antiplatelet and/or anticoagulant therapies for thrombotic causes

Live vaccines include:

►Influenza (intranasal) ►MMR ►Yellow fever ►Varicella ►Cholera ►Rotavirus (oral) ►Typhoid (oral)

CONTRAINDICATIONS of immunizing:

►Temporary (generally for live vaccines only) -Immunosuppression, Pregnancy Recent dose of IG: Depends on type and dose given ►Anaphylactic reactions to vaccine or inactive ingredient ►Live or inactivated

What is the brand new vaccine combo of 2021?

►Vaxelis (Merck and Sanofi Pasteur) — DTaP + Hib + IPV + Hep B Some combinations are indicated only for certain ages or certain doses in a series; consult package inserts

Ziconotide (Prialt)

"for the management of severe chronic pain in patients for whom intrathecal (IT) therapy is warranted, and who are intolerant of or refractory to other treatment, such as systemic analgesics, adjunctive therapies or IT morphine." Dose: Start with up to 2.4 mcg/day, titrate 2-3x/week up to a max 19.2 mcg/day

Chronic Non-Cancer Pain (CNCP) is an increasing trend in pain-related visits that affects >100 million adults annually in the United States. What are the cost considerations?

$560-635 billion annually on chronic pain -Direct care -Loss of productivity costs $17.8 billion annually on pain medications -$3.6 billion on opioids (20.1%) -$1.9 billion on NSAIDs (10.6%) $19,182 per patient per year -Direct health care and productivity costs -Increased costs with disability

Osteoarthritis Guideline:

++ Strongly recommend + Conditionally recommend -- Strongly recommend against - Conditionally recommend against NR No Recommendation

Egg based vaccines (chicken eggs)

- A/Guangdong- Maonan/SWL1536/2019 (H1N1) pdm09-like virus -A/HongKong/2671/2019 (H3N2)-like virus - B/Washington/02/2019- like virus (B/Victoria lineage) - B/Phuket/3073/2013-like virus (B/Yamagata lineage)

Epidemiology of rheumatoid arthritis:

-1% prevalence of the population -Prevalence increases with age up to 70s -3 times more common in women (6:1 in 15-45 age bracket) -Genetic predisposition -Environmental exposure

NAS Pharmacologic Treatment: Opioids Tincture of opium:

-25x higher conc. of morphine than PO morphine solution -Inc. risk of med errors when dosing/diluting product -Inc. risk of morphine overdose

What are the FDA approved 1st line therapies for Disease-Modifying Therapy?

-3 interferon (IFN) formulations (5 brand names) -Glatiramer acetate (non-IFN) Treatment decisions based on -Clinical predictors of disease severity -Mechanism of action -Safety and tolerability profile Use early in course of disease - beneficial Can be considered immunomodulatory or immunosuppressive

Cell or Recombinant Based

-A/Hawaii/70/2019 (H1N1) pdm09- like virus -A/HongKong/45/2019 (H3N2)-like virus - B/Washington/02/2019- like virus (B/Victoria lineage) -B/Phuket/3073/2013-like virus (B/Yamagata lineage)

Cancer pain treatment options?

-APAP, NSAIDs, Opioids, adjuncts -Bone pain ▪Bisphosphonates ▪Radiopharmaceuticals (stronium-89, samarium) ▪Corticosteroids Analgesics should be given "around the clock" rather than "on demand"

Interactions between genes, environment and epigenetics increase what?

-Abnormal immune response -Autoantibodies immune complexes -Inflammation -Damage

Systemic CAIs Reduce aqueous IOP by 25-40%, Decreases aqueous humor inflow and Used in those who fail maximal topical therapy - third line. What are the agents included?

-Acetazolamide (Diamox) -Methazolamide Do not combine with topical CAI

SLICC Clinical Criteria

-Acute/subacute cutaneous lupus/malar rash/photosensitive rash -Chronic cutaneous lupus/discoid rash -Oral OR nasal ulcers -Nonscarring alopecia -Arthritis/synovitis or tenderness -Serositis (pleuritis, pericarditis) -Renal (urine-to-creatinine ratio [or 24-hour urine protein] representing 500-mg protein/24 hr OR red blood cell casts) -Neurologic (seizure, psychosis, mononeuritis multiplex, myelitis, peripheral or cranial neuropathy, acute confusional state) -Hemolytic anemia -Leukopenia (<4,000/mm3) or lymphopenia (<1,000/mm3) -Thrombocytopenia (<100,000/mm3)

Tramadol (Ultram®)-- Labeling includes opioid boxed warning such as what?

-Addiction, abuse, misuse, death -Respiratory depression -Risk increases in combo with Benzo's and alcohol SE - nausea, constipation, drowsy Use caution in patients taking medications that lower seizure threshold (tricyclic antidepressants, first generation antipsychotics)

NAS Pharmacologic Treatment: Clonidine Central α2-adrenergic receptor agonist that reduce autonomic symptoms. What are the clinical trial data regarding this medication?

-Addition of clonidine reduces total morphine dose -Shortens treatment duration -Shortens length of stay No significant hypotension or bradycardia Requires extemporaneous preparation

Risk factors for POAG:

-Age / Family history -African descent -Myopia -Elevated IOP Less established: HTN, T2DM, Hypothyroidism, Migraine, Sleep apnea

Opioid-induced hypogonadism" Opioids inhibit release of gonadotropin-releasing hormone from the hypothalamus leading to ↓ testosterone & cortisol. What results in women?

-Alopecia -Amenorrhea -Depression -Reduced analgesic effect

The following characteristics below describe which medication? -Influenza A only -Resistance is common - Not recommended

-Amantadine and rimantidine

Adjuvants/Coanalgesics are often utilized for chronic pain and/or neuropathic pain have various classes of agents:

-Antidepressants - SNRI, TCAs -Anticonvulsants -Topical agents "add-on" therapy, or sometimes alone Not "classified" as analgesics but used for pain

SLICC Immunologic Criteria

-Antinuclear antibody (ANA) -Anti-double-stranded DNA (dsDNA) -Anti-Sm -Antiphospholipid antibody (lupus anticoagulant, anticardiolipin, anti-β2-glycoprotein I, false positive RPR) -Low complement (C3, C4, CH50) -Direct Coombs test (without hemolytic anemia)

NSAIDs can inc. lithium and methotrexate levels and inc. bleeding risk with concurrent use of what?

-Antiplatelets (clopidogrel, prasugrel, ticagrelor) -Anticoagulants (warfarin, dabigatran, apixaban, rivaroxaban) -Corticosteroids -Certain herbals (ie, ginkgo biloba) -Antidepressants (SSRIs, SNRIs)

Influenza regarding Controlling an outbreak:

-Antiviral chemoprophylaxis for at least 2 weeks -Continue 1 week beyond last known case is identified -Recommended for all residents: regardless of immunization status -Recommended for all unvaccinated employees

For Antithrombotic Prophylaxis assess risk for bleeding and for venous thromboembolism (VTE) as well as give medications 10-14 days and up to several weeks after. What medications should be given?

-Aspirin -Direct oral anticoagulants (DOACs) Rivaroxaban, apixaban, dabigatran -Low molecular weight heparin Enoxaparin, dalteparin -Low dose unfractionated heparin -Intermittent pneumatic compression

Treatment Overview for Ages 4-5: Methylphenidate if:

-Behavior interventions did not give significant improvement -moderate to severe disturbance in the child's function Behavior therapy shows positive improvement °Parent training in behavior management (PTBM) °Behavioral classroom interventions

Medication Treatment Options Decrease aqueous outflow:

-Beta blockers -- 1st Line -Alpha2 agonists -Carbonic anhydrase inhibitors

Alpha2-Adrenergic Agonists Decrease aqueous humor production with IOP reduction: 18-27% at 2-5 hours & 10% at 8-12 hours. What agents are involved:

-Brimonidine (Alphagan) 0.2% Alphagan P 0.15% -Apraclonidine (Iopidine)

Extraarticular Involvement of Rheumatoid arthritis includes blurred vision, vasculitis that can progress to ulcers, neurological symptoms and nodules (20% of pts). What are characteristics of nodules?

-Bumps on joints -Most common on the extensor surfaces of the elbows, forearms, and hands -Asymptomatic Others: Cardiac, Pulmonary, Lymphadenopathy (swollen), Splenomegaly, Felty Syndrome (RA, splenomegaly and neutropenia, resulting in susceptibility to bacterial infections)

Prevalence of Psoriasis:

-CA and US highest -About 2% of the population or 17 million people -Affects males & females equally -Most patients (~ 75%) have onset before the age 40 -Studies report two peak ages of onset: at 20 to 30 and 50 to 60 years

Which of the following are true about Gait Difficulties and Spasticity?

-Caused by spasticity, weakness, ataxia, defective proprioception, or combo of these -Spasticity often presents late in disease -Encountered commonly and tends to affect legs> arms -Can result in falls -Caution with muscle relaxants - don't want to make too weak

Non-pharm that really works: Lets start here by asking, "why is the immune system so upset?" When in doubt, look at the gut -- 5Rs

-Chances are dysbiosis/lifestyle-induced IP/leaky gut -SAD -Remove, Replace, Re-inoculate, Repair, Rebalance -Elimination diet -Probiotic

KNOW Environmental triggers can initiate the disease in genetically predisposed individuals INCLUDE:

-Cigarette smoke (hydrazine) -Ultraviolet light -Epstein-Barr Virus infection -Psychological stress -Silica dust -Solvents, petroleum, and dyes

BBW indicated with Rituximab (Rituxan) includes?

-Deaths have occurred, typically first infusion -Mucocutaneous reactions: fatal reactions have occurred -Hepatitis B virus reactivation -Progressive multifocal leukoencephalopathy Precaution/warning -Adverse cardiac effects (v. fib, MI) -Lymphopenia, leukopenia, neutropenia, thrombocytopenia, and anemia

Response to treatment of acute exacerbations of MS commonly seen within days, while evaluation of therapeutic outcomes conducted over months to yrs involve what?

-Decreased MS exacerbations -Decreased hospitalizations -Slowed disease progression and disability Important to recognize that over short term, little-no benefit may be seen

Atomoxetine (Strattera) common AE:

-Decreased appetite -Somnolence -Nausea/vomiting -Dry mouth -Palpitations

Using suspension or gel rather than solution decreases systemic exposure and side effects. Nasolacrimal occlusion does what?

-Decreases systemic absorption -Slight pressure to the inner corner of the eye -Hold for 1-5 minutes after administration -Decrease side effects, increase effectiveness DOT - Don't Open Eyes Technique -1-5 minutes for similar effect

The mechanism of Tissue Damage is unclear, however Immune complexes form between bound autoantibodies and nuclear material in end-organ tissues. What else results?

-Deposited complexes activate complement cascade leading to influx of inflammatory cells and tissue injury -Autoantibodies may directly react with proteins in the tissues -Antibodies against blood cells cause various cytopenias -Antiphospholipid antibodies lead to thrombosis and fetal death

Interferon adverse effects

-Depressed cell counts -Liver enzyme elevations: Transient; Respond to therapy DC -Injection-site redness: Necrosis - rarely -Swelling -Menstrual irregularities -Flu-like symptoms - fever, chills, myalgias -Depression: suicide risk

Tertiary symptoms relate to the effects of the disease on pts everyday life and complications resulting from primary symptoms are secondary. What results in primary symptoms?

-Direct consequence of conduction abnormalities produced by demyelination and axonal damage -Reflect the area of CNS where damage has occurred

Topicals: Methyl salicylate has many otc formulations, what are counseling points regarding this med?

-Do not apply to open wounds/damaged skin -Risk of 1st-3rd degree burns, especially in patients with neuropathic damage

Which of the following are true regarding Glatiramer acetate (Copaxone, Glatopa) -- 1st line Disease-Modifying Therapy ?

-Dosage - 20mg SC daily or 40mg SC TIW -Preferred agent in pregnancy -May be kept at room temp for up to 1 month (don't refrigerate after) -- do not freeze -Synthetic polypeptide consisting of L-alanine, L-glutamic acid, L-lysine, and L-tyrosine -Reduces inflammation, demyelination, and axonal damage -May also suppress T-cell activation -May be associated with neuroprotective effect

Systemic anticholinergics when causing pupil dilation and cycloplegia:

-Drugs for overactive bladder: tolterodine, oxybutynin -Drugs for Parkinson's: benztropine, trihexyphenidyl -Antihistamines -Scopolamine -Benzodiazepines -Phenothiazines -Tricyclic antidepressants and SSRI's

Progressive multifocal leukoencephalopathy (PML) seen in Natalizumab (Tysabri) is a rare brain infection most commonly seen in pts w/ HIV. Estimated risk for PML is low, what are the risk factors associated?

-Duration of treatment - 24 months or longer -History of John Cunningham virus (JCV) infection -Prior use of immunosuppressive therapies 271 cases have been identified - all on 8 months or longer Pts must be screened at baseline and every 6 months with a JCV test while on treatment

Azathioprine (AZA) is a purine analog metabolized to mercaptopurine, inhibits nucleic acid synthesis, cellular, and humoral immune functions. What are its indications?

-Early onset or mild-moderate lupus nephritis -Long-term maintenance therapy for lupus nephritis following cyclophosphamide induction -In place of steroids or in combination with steroids in more severe disease

Treatment Overview for Ages 12-16: FDA approved w/ adolescent

-Encouraged to prescribe behavior therapy -Assess for substance abuse -Re-assess for ADHD without substance use -Monitor for misuse and diversion °Consider non-stimulants °Stimulants with lower abuse potential

Opioid-induced hypogonadism" Opioids inhibit release of gonadotropin-releasing hormone from the hypothalamus leading to ↓ testosterone & cortisol. What results in men?

-Erectile dysfunction -↓ libido -Reduced analgesic effect

Hyperactivity/ Impulsivity

-Excessive talking -Fidgeting -Inability to remain seated in appropriate situations -Often runs about excessively at inappropriate times -Often blurts out answers before a question completed -Often interrupts or intrudes on others

Physical therapy/Occupational therapy assist with:

-Exercise is more effective when supervised -Self-efficacy and self-management training -Joint stability, mobility, range-of-motion -Assistive devices - canes and walkers -Joint protection, energy conservation -Activities of daily living and assistive devices -Heat

Cardiovascular Safety

-Family history of cardiac problems °Sudden cardiac or unexplained death under age 40 in first-degree relative °Significant cardiovascular concerns ±Frequent syncope, especially exercise induced ±Excessive shortness of breath or exercise intolerance -Use caution with stimulants and atomoxetine -Monitor closely

Disease-Modifying Therapy Inadequate response or intolerance to first line:

-Fingolimod -Natalizumab -Mitoxantrone

Switching from once daily methylphenidate, use only ½ as much amphetamine

-Focalin (dexmethylphenidate) & Metadate CD 20 mg/day should start with 10 mg/day of Adderall XR -If the once daily medication wears off too soon, a dose of IR may be added or switch to once daily that lasts a little longer: ±Adhansia XR or Concerta for methylphenidate ±Mydayis or Vyvanse for amphetamines

Secondary-progressive MS (SPMS)

-Follows RR-MS -Most pts with RR-MS will progress to this -After period of relapses and remissions, will start to steadily progress -May or may not continue to have remissions

The main AE of Methotrexate are THROMBOCYTOPENIA & FOLATE DEPLETION others include what?

-GI: N/V/D, elevated liver enzymes -Leukopenia -Pulmonary Fibrosis and pneumonitis - rare

Risk factors for developing MS:

-Geography -Age on onset: 15-45y/o -Age at migration: <15 adopt risk migrated to -Genetics: (Higher risk with fam hx[twins 25% risk]) -Gender: (women > men -- 2:1) -Cigarette smoking (Inc. risk/more severe progression) -Infections: (viral or bacterial)

FDA-approved for combination with a stimulant:

-Guanfacine -Clonidine

Alpha Agonists: ER forms are less sedating than IR

-Guanfacine ER (Intuniv) -Clonidine ER (Kapvay) -Clonidine IR may be useful for sleep in patients with ADHD May be preferred in patients with history of substance abuse (no DEA schedule) AE: Somnolence & dry mouth; Guanfacine may cause moderate weight gain

Start treatment ASAP in the following:

-Hospitalized with influenza, regardless of duration prior to hospitalization -Outpatients of any age with severe or progressive illness, regardless of duration -Outpatients who are high risk for complications - chronic medical conditions or immunocompromised -Children <2 and adults ≥65 -Pregnant women + within 2 weeks postpartum

Treating to target helps to keep providers regularly following up with patients and reminds them to adjust therapy more frequently when clinically warranted. What are good approaches?

-Identify a target of therapy (remission, reduction in activity score, etc.) and a method for monitoring -Monitoring at predetermined intervals -A plan to change therapy if the target is not achieved at monitoring point -Collaboration and shared decision-making with the patient

Safety summary:

-In general live vaccines have more precautions and contraindications -In general live vaccines have more stringent storage requirements -In general 4 weeks is the minimum time between doses in multi-dose series -Proper injection technique is essential to maximize efficacy and avoid SIRVA -Be prepared for acute vaccine reactions

Biologic DMARDs are historically reserved for Tx failure (mod. to severe), expensive and is involved with less toxicities with less monitoring which results in what?

-Increase risk of infection -Must test for TB prior to initiation of therapy -If develop infection should discontinue until infection is resolved -No live vaccines should while taking biologics

Enzyme systems regulate purine metabolism: 2 main regulation abnormalities are what?

-Increased activity of phosphoribosyl pyrophosphate (PRPP) synthetase -Deficiency of hypoxanthine-guanine phosphoribosyltransferase (HGPRT)

Methadone has many concerns and generally used in cancer patients. What are the extreme caution points to watch for?

-Increasing number related deaths -QTc interval prolongation: serious arrythmias (TdP) -Complication (unclear) conversion to/from other opioids

Influenza Summary

-Influenza A and B cause human illness -Influenza vaccinations are essential for prevention of disease -Various treatments may be used to treat patients with influenza but questions exist about cost benefits

What are key characteristics regarding race for OA?

-Knee OA is twice as likely in black women than white -Lower incidence of OA in Chinese, East Indian, Native American compared to Caucasian

Pathophysiology for ADHD:

-Lack of connectivity between prefrontal cortex and parietal lobe impairs the ability to suppress the default mode when needed -Stronger short-range connection (limbic area) and reduced number of long-range connections (attention and control systems)

Prostaglandin Analogs: Analogs of prostaglandin Fα -25-35% reduction in IOP -Inc. uveoscleral and trabecular outflow of aqueous humor These agents include:

-Latanoprost (Xalatan) -Travoprost (Travatan Z) - uses diff preserve -Bimatoprost (Lumigan) - more eff. in some pts -Unoprostone (Rescula) -Tafluprost (Zioptan) -- preserve-free single use containers -Latanoprost / Timolol (Xalcam) - unopened in fridge Once daily dosing for these (1 drop) Apply, then wait 15 minutes before wearing contact lenses Exception - Unoprostone (Rescula) requires BID dosing

Progressive-relapsing MS (PRMS)

-Least common form -Steady disease progression and worsening function -Occasional relapses

Pregnant patients with lupus can expect exacerbations during pregnancy and in the early postpartum period. Thrombotic events and inflammatory changes can lead to spontaneous abortion, preeclampsia, premature birth. Treatment include what?

-Low-dose ASA ± subcutaneous heparin -Steroids -Hydroxychloroquine -Azathioprine (as a last resort) Avoid cytotoxic medications (azathioprine is best if one has to be used) during pregnancy

Corticosteroids are great for flares and adjunct (bridge therapy/combo) but not for monotherapy despite anti-inflammatory and immunosuppressive effects. What purpose do they serve?

-Low-dose glucocorticoid ≤10 mg/day of prednisone (or equivalent) -High-dose glucocorticoid >10 mg/day of prednisone (or equivalent) and up to 60 mg/day with a rapid taper -Short-term glucocorticoid <3 month treatment

Pathophysiology for Default mode network (DMN):

-Medial prefrontal cortex, medial parietal lobe (precuneus), and posterior cingulate -Active during resting state and suppressed during active attention

Dose depends on joint size for Intraarticular Glucocorticoids. What are the medications involved?

-Methylprednisolone 4-80 mg -Triamcinolone 10-40 mg -Extended-release triamcinolone (Zilretta) No proven benefit

High quality evidence supports use of Antimalarial Agents in all patients, including pregnant women, as hydroxychloroquine therapy improves long-term survival and prevents flares. Evidence shows what?

-Moderate evidence for protection against bone loss, thrombosis, and irreversible organ damage -Beneficial effect on lipids and fasting blood glucose -Decreased infections -Decreased risk of thrombosis in antiphospholipid patients

Anakinra (Kineret) Interleukin - 1 Receptor Antagonist, a Natural anti-inflammatory that is indicated for what?

-Moderate to severe active RA in patients 18 years and older who have failed 1 or more DMARDs -Neonatal-onset multisystem inflammatory disease

Tofacitinib (Xeljanz) Humanized monoclonal antibody that prevents IL-6 from interacting with the IL-6 receptor. This medication can be used monotherapy or in combination with methotrexate or other nonbiologic DMARDs. What is this medication indicated for?

-Moderate to severe rheumatoid arthritis, have failed other therapies, typically MTX or another biologic -Psoriatic arthritis: if inadequate response/intolerance to methotrexate or other DMARDs -Ulcerative colitis: moderate to severe

Atomoxetine Contraindications:

-Monoamine Oxidase Inhibitors w/ 2 weeks -Narrow angle glaucoma: mydriasis -Pheochromocytoma: Elevated BP and HR -Severe cardiovascular disorders AE: Liver injury, CV, Priapism, Urinary retention, BP/HR

POAG: Monitoring

-Monocular trial may be used -Initial follow-up after 4-6 weeks -After IOP goal reduction is reached, Q 3-6 months, at least annually -Visual field and disc changes - annually -Adherence and tolerance

NAS Pharmacologic Treatment: Opioids Morphine vs. methadone:

-Morphine is the most common medication used -Half-life •Methadone: longer & more variable (6 - 62 hours) •Morphine: shorter allowing for easier dose titration -No difference in treatment outcomes, length of stay -No dose/frequency been proven to be clinically superior

Relapsing-remitting MS (RRMS)

-Most common type -Inflammatory attacks on myelin and the nerve fibers Worsening neurological function -Symptoms flare unexpectedly (relapse), then disappear(remission)

Chronic Non-Cancer Pain (CNCP) have similar treatment principles but much more complex in regards to what?

-Multiple modalities utilized -Multidisciplinary -Closely aligned with treatment of comorbid conditions (diabetes, depression, etc.) -Treatment choices vary with type of pain (Neuropathic vs. visceral/somatic) -Limited data on the benefit/safety of opioids

HepB IM is routine for all infants and should vaccinate adults if risk factors are present or for any person for whom immunity against hep B is desired. What are risk factors?

-Multiple sex partners -Patients seeking treatment for STDs, HIV infection -Injection drug users -Patients on dialysis -Chronic liver disease -Health care providers -diabetes 19 to 59 years old (permissive if 60 years and older) ACIPrecommendationisforalladultpatientswithdiabetesaged19through 60years. Frequently asymptomatic until too late

Angle Closure Glaucoma often triggered by dim light, emotional stress, dilating drops leading to what?

-Mydriasis (dilation) can cause pupillary block -Pupillary block - the lens seals against the iris blocking outflow Red eye, mid-dilated or oval pupil, cloudy cornea

AE of Leflunomide (Arava) are dose related and includes:

-N/V/D -Alopecia -Bone marrow suppression -CBC monthly x 6 months, then q 6-8 weeks -Contraindicated in liver disease -LFTs >2x ULN = Stop -Monitor LFTs monthly x 3 months then quarterly -Pregnancy Category X Monitor TB at baseline and preg. test prior to initiating

Aspirin (ASA) regarding CV risk when used for cardioprotection:

-NSAID MAY interfere with ASA activity -Dose NSAID 8 hours before ASA OR -Dose NSAID 30 minutes after immediate-release ASA -All NSAIDs have potential to interfere with antiplatelet effect -Consider use of PPI for GI protection

Pharm therapies for RA include what?

-NSAIDs -Corticosteroids -Disease-modifying antirheumatic drug=DMARD TNF-α inhibitors Non-TNF biologics Tofacitinib

Disease-Modifying Therapy Treatment of relapsing forms of MS*:

-Natalizumab -Fingolimod -Teriflunomide -Dimethyl fumarate -Ocrelizumab (Ocrevus)

What are the AE of Mitoxantrone (Novantrone)?

-Nausea -Alopecia -Menstrual disorder -Amenorrhea -Upper respiratory tract infection -UTIs -Leukemia

Disease-Modifying Therapy Progressive or worsening MS:

-Ocrelizumab (Ocrevus) -Mitoxantrone

Alpha2-Adrenergic Agonists -Adverse Effects:

-Ocular - burning, stinging, blurring, allergic-like reaction, pruritis, edema of lid and conjunctiva, foreign body sensation -Systemic - dizziness, fatigue, somnolence, dry mouth, potential BP and pulse reduction Use caution CV disease, renal insufficiency, diabetes, cerebrovascular disease, antihpertensives, MAOI, tricyclic antidepressants

somatic nociceptive pain:

-Origin: musculoskeletal, joint, or skin damage -Location: easy to pinpoint -Description: sharp, intense Ex: laceration, shoulder/knee injury, gout, osteoarthritis

Visceral nociceptive pain:

-Origin: organs or smooth muscle damage -Location: hard to pinpoint and may radiate to other areas -Description: dull, vague, aching, squeezing Ex: IBS pain, menstrual cramps, appendicitis, gallstones, heartburn, myocardial infarction

Consider treatment in the following:

-Outpatients with illness <48 hours -Outpatients who are household contacts of those at high risk for developing complications -Healthcare providers caring for patients who are high risk

Bowel and Bladder Symptoms refers to Incontinence, urgency, frequency, nocturia and Indicate hyperreflexic bladder. What Anticholinergic agents (mild) are used for these symptoms?

-Oxybutynin (Ditropan - 10-20mg/day) -Tolterodine (Detrol - 2-4mg/day) -Propantheline (Pro-Banthine - 45-90mg/day) -Hyoscyamine (Levsin - 0.75-1.5mg/day) -Dicyclomine (Bentyl - 30-80mg/day) Caution with falls, decreased cognition, and constipation

Nonpharmacologic Interventions:

-Patient and caregiver education about the condition and treatments -Behavioral modification -Individualized educational plan (IEP) -Change homework to smaller segments of work -Use a planner -Cognitive behavioral therapy -Behavioral contract

DSM 5 overview

-Pattern of behavior present in 2 or more settings that results in performance issues related to social, educational, and work performance -Children - 6 symptoms -Adults and adolescents - 5 symptoms -Symptoms present for at least 6 months -Several symptoms occur prior to age 12

Age 4-5 Medication Concerns: Fewer studies in this age group Decision to prescribe:

-Persistence of symptoms for 9 months -Dysfunction occurs in more than one setting -Lack of adequate response to behavior therapy Concerns about growth slowing limited to moderate-severe dysfunction

Parasympathetic Increase aqueous humor trabecular outflow. Ciliary muscle contraction pulls open the trabecular meshwork. Use is limited by local side effects and frequent dosing. Agents included are:

-Pilocarpine (Isopto Carpine, Pilocar, Pilopine HS) -Carbachol (Carboptic, Isopto Carbachol)

Use Caution in BB with:

-Pulmonary disease -Sinus bradycardia -2nd or 3rd degree heart block -Congestive heart failure -Diabetes -Oral beta-blocker therapy

Prostaglandin Analogs tend to have Iris pigmentation, hypertrichosis, eyelash pigmentation but also have fewer systemic AE's than BBs. One should D/C due to local effects (10-25%) due to:

-Punctate corneal erosions -Conjunctival hyperemia

Alemtuzumab (Lemtrada) Only indicated if pt has inadequate response to 2 or more drugs and CI in pts with HIV. What are its AE?

-Rash -Headache -Fever -Fatigue -Insomnia -Urticaria -N/V/D -Flushing Complete all vaccinations 6 wks before therapy

Intraarticular Glucocorticoid are preferred over other injections, what are other criteria regarding this medication?

-Recommended for hip and knee, may work for hand -No more than every 3 months or 3-4 times per year -Onset in 1-2 days and benefit may decline after 1 month -Aspiration of effusion may lead to placebo effect -Ultrasound guided for hip -May add local anesthetic (Bupivacaine, lidocaine)

Goals of Therapy of intraocular pressure (IOP):

-Reduce by 20-30%: Preferred 25-30% -Prevent changes to visual field, optic disc, & retinal nerve fibers -Minimize AE's -Promote medication adherence

Sustained Release Stimulants:

-Reduced stigma for kids: no dose needed at school -Reduced risk of abuse -Improved adherence -Pharmaceutics of the dosage forms -Bioequivalence of generic products

Chronic Rhinosinusitis/Nasal Polyps Treatment in CF:

-Saline nasal irrigation -Topical anti-inflammatory agents -Surgery

INFLUENZA COMPLICATIONS usually pts older than 50 and young children as well as immunocompromised pts.:

-Secondary bacterial pneumonia, sinusitis, or otitis -Exacerbate underlying pulmonary or cardiac disease -Encephalopathy -Myositis, myocarditis, pericarditis -Death: 2-fold increase in fatal myocardial infarction and ischemic heart disease

Alemtuzumab (Lemtrada) Boxed warnings include what?

-Serious, sometimes fatal autoimmune conditions -Serious infections -Infusion reactions -Malignancies

IV methylprednisolone

-Shown to shorten duration of exacerbation -May also delay repeat attacks -Oral equipotent doses of prednisone may be substituted -Doses from 500-1000mg daily -- 3-10 days -Recovery faster if drug started within 2 weeks of symptom onset -Improvement within 3-5 days

Nociceptive Pain is caused by damage to tissues or organs detected by nociceptors; protective. What are 2 types of Nociceptive Pain?

-Somatic -Visceral

Inactivated influenza vaccine four strains (IIV4)

-Standard-dose—Egg-based (IIV4s) -Standard-dose—Adjuvanted Egg-based (aIIV4) -Standard-dose—Cell culture-based (ccIIV4) -High dose—Egg-based (HD-IIV4) Recombinant influenza vaccine (RIV4) - army worm Live attenuated influenza vaccine (LAIV4) - weakened nasal spray

"Pain Ladder" - WHO Developed for cancer pain, in practice stepwise approach is appropriate for all pain:

-Start with non-opioids -Utilize adjuvants whenever possible -Add opioids if pain persists

Pre-medicate (methylprednisolone/diphenhydramine) prior to each infusion for Ocrelizumab (Ocrevus). What are dosing regarding IV infusion?

-Starting dose: 300mg IV infusion, followed 2 wks later by 2nd 300mg IV infusion -Subsequent doses: 600mg IV infusion every 6 months -Must be diluted prior to administration -Monitor pts closely during and for at least one hr post infusion

Primary progressive MS (PPMS)

-Steady worsening of neurological function No relapses or remission -Overall progression is continuous -Age of onset ~ 10 yrs after RR MS -Male = female -Worse prognosis vs RRMS

Beta Blocker AE:

-Stinging -Local - dry eyes, corneal anesthesia, blepharitis, blurred vision -Rarely - conjunctivitis

Clonidine ER (Kapvay) Patient education:

-Swallow whole, do not crush or cut or chew - ER tab -If prone to syncope, avoid dehydration and overheating -Avoid other sedatives and be careful driving -Do not discontinue abruptly: BP rebound increase Do not substitute with other clonidine products

What to do in case of Histamine-related side effects (urticaria, pruritis, bronchospasm, , flushing, itching, hives, sweating) ?

-Switch to an opioid with less histamine release -Tx: Consider antihistamine (ie, diphenhydramine, cimetidine) Meperidine, Codeine, Morphine, have highest release With true allergy, can generally consider agent of different opioid class with caution [Severe hypotension, rash, breathing/swallowing difficulty, swelling of throat]

Intraarticular - Hyaluronic Acid refers to proteoglycans in cartilage that are bound into a collagenous mesh and function to attract and maintain water molecules. Visulosupplementation is needed with such agents, What are the agents involved?

-Synvisc® 16 mg (2 mL) once weekly X 3 -Hyalgan® 20 mg (2 mL) once weekly X 5 -Euflexxa, Supartz, Orthovisc

NSAID Dosing/Counseling

-Take with food to reduce stomach upset -Can increase your risk of bleeding -Avoid alcohol -May need to stop taking before surgery -Avoid using if you have heart disease -Do not use if you are pregnant

Cartilage damage is changes in the size and arrangement of collagen fibers in the extracellular matrix. What is the association with chondrocytes?

-They attempt to repair the damage -They lose the ability to balance between breakdown and resynthesis of cartilage with an increase in apoptosis Loss of cartilage leads to joint space narrowing

Medications used acutely for AACG:

-Topical beta-blocker -Topical alpha-2 agonist -Topical or systemic carbonic anhydrase inhibitor -Topical miotic Systemic hyperosmotic: -Mannitol 1-2 g/kg IV: 15-25% solution over 30-60 minutes -Hypertonic saline 3mL/kg IV: 3% soln over 20-30 min

What are the General Pain Principles?

-Treat pain at initial onset for best efficacy -Utilize a "multimodal" approach (ie, analgesics with different MOAs, non-pharmacological therapies) -Treat with lowest dose that adequately reduces pain

Risk factors for CVD risk include:

-Unstable angina -History of MI -CABG surgery -Ischemic stroke -High cumulative CV risk

CI of Stimulants:

-Use of an MAOI within 14 days Inc. HR & BP leading to serious cardiovascular risk -Hypersensitivity angioedema and anaphylactic reactions

In an impaired joint, bone growth (osteophyte or bone spur) alters the contour of the joint and reduces mobility. What else occurs?

-Vertical clefts develop -Areas of repair develop but function is inferior -Synovitis leads to thickening of the joint capsule Periarticular muscle wasting is common

Common therapies associated with Complementary and Alternative Medicine involve diet and dietary supplements. Antioxidant supplements include what?

-Vitamin A, C, E, -Alpha-lipoic acid -Coenzyme Q10 -Grape seed -Pine bark extracts -Mangosteen -Acai suggestive evidence for benefitting MS, but risk because these types of supplements can stimulate the immune system Ultimately, insufficient evidence supporting these, but may be considered

The three main factors affecting drug distribution in neonates, infants and children are what?

-water and fat distribution (Volume) -protein binding -permeability of the blood brain barrier

Sulfasalazine is indicated for all disease durations and severities. What are the dosing considerations with this medication?

0.5 - 1 gm PO daily x 1 week [enteric coated] Inc. daily dose by 500mg q week until 2 gm TDD (3 gm max) Can titrate more slowly or divide dose to minimize AE

RECOMMENDATIONS FOR 2-64 HIGH-RISK with the associated problems below: Smoker, heart/liver/lung, disease, diabetes, alcoholism

1 dose of PPSV23 + 1 dose of PPSV23 at age 65

Guanfacine ER (Intuniv) (1 mg, 2 mg, 3 mg, 4 mg extended-release tablets)

1 mg daily inc. by 1 mg/day increments after at least one week °Target dose: 0.05-0.12 mg/kg/day (Max of 7 mg once daily) Hypotension, Bradycardia, Syncope: Titrate and Taper Morning or evening dosing: Taper no more quickly than 1 mg/day decrease every 3-7 days

TREATMENT WITH ANTIVIRALS Shorten the duration of the flu by ________ days

1-2 (clinicians make decision if its worth it) Limited evidence that they prevent complications, hospitalizations or deaths Prophylaxis can prevent symptomatic disease, but does not decrease virus transmission

Maintenance Fluids 1. Using the Holliday- Segar Method, calculate the maintenance fluid requirement for a 13 kg patient. 2. Using the Holliday- Segar Method, calculate the maintenance fluid requirement for a 45 kg patient.

1. 11 - 20 kg = 1000 mL/day + 50 mL/kg/day for > 10 kg 1000 mL/day + 50 (3 kg) mL/day = 1150 mL/day 2. > 20 kg = 1500 mL/day + 20 mL/kg/day for kg > 20 1500 mL/day + 20 (25 kg) mL/day = 2000 mL/day

Protectint against influenza A and B viral iinfection CDC stated no preference is expressed for any influenza vaccine over another however. Match the following terms to their correct terminology: 1. Egg based 2. Adjuvanted 3. Cell culture 4. Recombinant

1. "grown in chicken eggs" 2. MF59 an oil-in-water emulsion of squalene oil Cell 3. grown in canine kidney cells 4. use virus from the ovaries of the fall armyworm in the pupal stage using recombinant DNA technology

Match the following EFFICACY COMPARISON according to their correct terms: 1. Influenza 2. COVID-19 3. MMR 4. PPSV-23

1. 39% (range 40-60%) 2. 94-95% (after both doses) 3. 97% measles (after both doses) 4. 60-70%

Tofacitinib (Xeljanz) is dosed in both extended and immediate release. Match the following terms to their correct dosages. Not recommended to combine with biologics or potent immunosuppressants (azathioprine, cyclosporine) 1. Immediate release: 2. Extended release:

1. 5 mg PO twice daily 2. 11 mg PO once daily

Match the following characteristics below for the following med: Oseltamivir (Tamiflu®) -- wt based birth-12; >/=2wks 75mg BID within 48 hrs of symptoms; treat 5 days 1. Adult Prophylaxis: 2. Child Prophylaxis:

1. 75mg daily within 48 hrs of contact with infected person; treat 7 days 2. 3 months - 12 years: Weight based *FDA approved: 1 yr. & older -Take with food -No current shortage; can compound capsules into suspension -Dose adjust for renal impairment

Medications that can contribute to formation of psoriasis include:

1. ACEIs 2. NSAIDs 3. Propanolol/Inderal 4. Lithium 5. Salicylates 6. Steroid withdrawal 7. Beta blockers 8. Synthetic anti-malarials 9. Tetracyclines 10. Interferons

Common Stimulant Side Effects:

1. Abdominal pain: W/ full stomach, Lower dose 2. Headaches: Divided doses, W/ food, Ibuprofen/acetaminophen 3. Appetite loss: High cal breakfast, Cyproheptadine 4. Sleep disturbance: Last dose earlier or lower, Choose non-stimulant, Melatonin

Match the following non-Opioids to their specific coverages: 1. Analgesic, antipyretic, but NOT anti-inflammatory Prostaglandin modulation May be used 1st line in some disease states (ie, osteoarthritis) 2. Analgesic, antipyretic, but also anti-inflammatory Prostaglandin inhibition: decreased pain impulses received in CNS Used for MANY disease states (product dependent)

1. Acetaminophen (APAP) 2. NSAIDs

1st line agents for moderate-severe psoriasis include:

1. Acitretin (oral retinoid) 2. Cyclosporine 3. Methotrexate 4. Biologics 2nd line: tacrolimus, mycophenolate, pimecrolimus, sulfasalazine, hydroxyurea, 6-thioguanine*

Match the following to tapering methods below to the correct terminology: 1. Decrease dose by 20% daily 2. Decrease dose by 10% q 3-5days

1. Acute opioid use 2. Chronic opioid use

Risk factors for OA include:

1. Age 2. Female sex 3. Race 4. Obesity 5. Genetic factors 6. Major joint trauma 7. Repetitive stress 8. Congenital/development defects 9. Prior inflammatory joint disease 10. Metabolic/endocrine disorders

Match the following calcineurin inhibitors with their FDA approved ages. 1. Tacrolimus ointment /Protopic 0.03% 2. Tacrolimus ointment/Protopic 0.1% 3. Pimecrolimus cream/Elidel 1%

1. Ages 2-15 y/o 2. Ages ≥16 y/o 3. Ages ≥2 y/o

Match the following definitions to their terms: 1. Pain 2. Acute Pain 3. Chronic Pain

1. An unpleasant sensory and emotional experience associated with actual or potential tissue damage or described in terms of such damage. 2. Pain with a quick and sudden onset (cause is usually known), lasting for a shorter period of time (i.e. until the injury or disease is healed). 3. Persistent pain (continuous or recurrent) that lasts beyond the anticipated healing time or for more than 3 to 6 months and adversely affects the patient's well-being, level of function, and quality of life.

Medium & low potency corticosteroids:

1. Betamethasone valerate (medium) 2. Hydrocortisone valerate (medium) 3. Triamcinolone acetonide (medium) 4. Hydrocortisone (low/least potent)

Investigational biologic agent considerations:

1. Blisibimod - inhibits soluble & membrane BLyS 2. Atacicept - blocks both BLyS & APRIL (a proliferation-inducing ligand)-mediated B cell stimulation 3. Sifalimumab & anifrolumab - target type 1 interferons

Pathophysiology of LUPUS:

1. Characterized by disorders of innate & adaptive immune systems 2. Increased quantities and immunogenicity of nucleic acids and self-antigens induce activation of innate immunity and formation of auto-antibodies and T cells 3. Interactions with genetic and environmental factors lead to increased immune response and subsequent inflammation and destruction

Examples of psoriatic triggers include:

1. Cold weather 2. Dry weather 3. Alcohol 4. Skin injury (trauma, sunburn) 5. Smoking 6. Stress (infection) 7. Medications

Treatment strategies of RA:

1. DMARD Monotherapy 2. Combination therapy -Dual Methotrexate plus hydroxychloroquine Methotrexate plus leflunomide Methotrexate plus sulfasalazine -Triple DMARDs Sulfasalazine, hydroxychloroquine, and methotrexate

Guanfacine ER pt Education:

1. Do not crush or chew or break tablets because of a dangerous release of the full dose 2. Do not take with a high-fat meal because of an increased effect of the drug 3. Do not stop taking the medication without decreasing gradually 4. Sedation is important to monitor especially at the start of treatment 5. Avoid becoming dehydrated or overheated 6. Do not use alcohol

GI effects most often seen with NSAIDs include:

1. Dyspepsia 2. Heartburn 3. Abdominal pain 4. Nausea 5. Diarrhea 6. Ulceration 7. Bleeding *mechanism is both direct & indirect*

1st line agents for Mild-moderate psoriasis include:

1. Emollients 2. Keratolytics 3. Topical corticosteroids 4. Vitamin D analogues 5. Tazarotene (topical retinoid) *2nd line: coal tar, anthralin, immunomodulators*

Match the following common mutations with their perspective class: 1. Class I: 2. Class II: 3. Class III: 4. Class IV: 5. Class V: 6. Class VI:

1. G542X 2. F508del 3. G551D 4 R117H 5. Reduced CFTR synthesis 6. Reduce the half-life of functional CFTR in the cell membrane = reduced quantity of CFTR

Predisposing factors of psoriasis include:

1. Genetics 2. Genes 3. Skin trauma, skin disorders & infection 4. Alcohol consumption 5. Smoking 6. Obesity 7. Psychological stress 8. Medications

Non-Pharmacologic Recommendations:

1. Good social support 2. Aerobic exercise may reduce risk for cardiovascular events and osteoporosis and improve symptoms 3. Increased sun protection due to photosensitivity 4. Smoking cessation

High potency corticosteroids include:

1. Halobetasol propionate (very high) 2. Betamethasone diproprionate ointment (very high) 3. Fluocinonide (high) 4. Triamcinolone (high) 5. Betamethasone diproprionate cream (high)

____1_____ develop on the end joints of the fingers (distal interphalangeal) while ____2____ develop on the middle joints of the fingers (Proximal interphalangeal).

1. Heberden's nodes 2. Bouchard's nodes

Risk factors for life threatening asthma:

1. History of previous life-threatening asthma exacerbation requiring ICU admission and/or endotracheal intubation 2. Denial of, misunderstanding of, or failure to perceive severity of illness 3. Poor asthma control, noncompliance 4. Lack of social support 5. Associated psychiatric disorder 6. African American or Hispanic descent

Match the following CI allergies to the the vaccines to avoid: 1. Eggs 2. Gelatin 3. Neomycin 4. Streptomycin 5. Polymixin B

1. Influenza (egg based), Yellow Fever 2. MMR, Varicella 3. MMR, Varicella, Polio 4. Polio 5. Polio False CI: Mild-moderate local reactions; Mild acute illness; Prematurity; Family history of adverse events; Diarrhea; Breastfeeding; Current antibiotic use

Measuring Progression of MS is done by what?

1. Kurtzke Expanded Disability Status Scale (EDSS) -Rate scale: 0-10 to evaluate neurologic function -Limitations: focuses on impairment of walking 2. Multiple sclerosis functional composite (MSFC) -Being evaluated for increased sensitivity and utility in describing changes in disability over time 3. MRI: Used as index of disease activity and progression 4. Optical coherence tomography (OCT) -Measures retinal neural fiber layer thickness -Measurable sign of pathological progression over time

Vaccine Side Effects 1. Live 2. Inactivated

1. Local injection site reactions; Mimic of mild case of disease 2. Pain at the injection site; Mild systemic symptoms False Side effects: Autism: no evidence Thimerosal: Mercury containing preservative, no evidence of autism

Stimulants include: (ALL C-2)

1. Methylphenidate IR: Ritalin, Methylin (oral solution) -ER: Concerta 2. Methylphenidate transdermal: Daytrana 3. Dexmethylphenidate: Focalin, Focalin XR 4. Dextroamphetamine/Amphetamine IR: Adderall -ER: Adderall XR, Mydayis 5. Lisdexamfetamine: Vyvanse 6. Amphetamine 7. Methamphetamine

1. Nucleic acid vaccines use DNA or RNA from a virus and insert into human cells: 2. Protein based- viral protein fragments or empty viral shells are injected directly into the human body to elicit a response:

1. Moderna (4wks apart) and Pfizer (3wks apart) are the 1st RNA vaccines 2. Human Papilloma virus- HPV vaccines

Pilocarpine (Isopto Carpine, Pilocar, Pilopine HS) is known to have Retinal tears, allergic reaction, worsening of inflammatory conditions. What are the Ocular AE associated?

1. Myosis - decreased night vision, decreased vision in patients with central cataracts 2. Headache, brow ache, periorbital pain, eyelid twitching, conjunctival irritation -Improves over 3-5 weeks Systemic - diaphoresis, N/V, diarrhea, cramping, urinary frequency, bronchospasm

*KNOW* Match the following corticosteroids to their dosage forms. 1. Prednisone or prednisolone 2. Methylprednisolone or triamcinolone 3. Intraarticular triamcinolone

1. Oral 2. IM 3. IA

Match the following corticosteroids: 1. Dosed at 0.5 mg/kg x2-5 days, then taped over 7-10 days 2. Must be followed by oral prednisone or prednisolone 3. Used in combo with NSAIDs, CCS or colchicine and ONLY used for 1-2 joints

1. Oral prednisone/prednisolone 2. IM Methylprednisolone or triamcinolone 3. IA triamcinolone

PCA pump dosing characteristics include the following below: 1. Bolus dose 2. Basal rate 3. Lockout interval

1. PRN for breakthrough pain 2. "around-the-clock" infusion 3. minimum time between "bolus" doses

Fat soluble vitamins: 1. K 2. A 3. D 4. E

1. Peripheral neuropathy, myopathy, hemolysis 2. Ocular (night blindness, conjunctival corneal xerosis), skin involvement (follicular hyperkeratosis) 3. Nutritional rickets, osteopenia, osteoporosis, fractures 4. Coagulopathy, bone disease

Match the following to the correct description: 1. Inactivated only in the US; Oral may cause vaccine associated paralytic poliomyelitis 2. Oral administration; only initiated in those <15weeks

1. Polio - IM, SQ 2. Rotavirus

Non-pharmacological techniques for management of airway secretions:

1. Postural Draining-patient is placed in a downward position allowing airway drainage with the aid of gravity 2. Percussion-clapping on the chest with a cupped hand loosens airway secretions and cause vibration-caregivers create vibrations that shake secretions from the walls of the lung 3. High frequency chest wall oscillation-done with a vest that is based off the patient's size 4. Active cycle of breathing or forcing oneself to cough coughing or deep breathing can loosen and expectorate mucus 5. Exercise-help get everything moving and out of the body

PNEUMOCOCCAL VACCINE Protects from Streptococcal pneumonia. Match the following below: 1. Conjugate Polysaccharide vaccine (PCV13): 2. Polysaccharide (PPSV23):

1. Prevnar 13 - IM 2. Pneumovax - IM, SQ [not be used in less than 2]

Approach to Care - Pediatrics Use information gathered from guardians, teachers, school and mental health clinicians. Make the diagnosis based on the DSM-5 criteria, symptoms in more than one setting as well as what?

1. Rule out alternative causes for the behavior problems 2. Assess other emotional and behavioral, developmental and physical concerns that may affect behavior 3. Follow a model of chronic care and medical home 4. Titrate doses of medication to achieve maximum benefit and minimum adverse effects

General Approach to Treatment for MS:

1. Severity of symptoms at onset -Determines if induction or escalation algorithm will be used 2. When FDA approved drugs fail to alter disease, investigational agents or non-FDA approved meds used -Rituximab 3. Most productive years of life -Realistic goals and expectations -Long-term treatment plan 4. Support system, psychological support 5. Nonadherence is an issue 6. COST

Match the following AE of Corticosteroids - methylprednisolone: 1. Short-term use: 2. Long-term use: 3. Pts with DM:

1. Sleep disturbance, Metallic taste, GI upset (rare) 2. Acne, Fungal infections, Mood alteration, GI hemorrhage (rare) 3. Sig. elevations of blood sugar (may need to use insulin)

General Dosing of Opioids

1. Start with lowest effective dosage and titrate; Latest data shows risk > benefit when MME >50mg/day 2. Opioid-naïve patients should generally start with IR opioids; Prescribe no greater quantity than needed 3. Long-acting opioids should be utilized for maintenance dosing for persistent, around-the-clock (ATC) pain; •PRN therapy using IR opioids can be added for break-through pain only (note, if using PRN therapy ATC: increase LA opioid)! 4. Re-evaluate within 1-4 weeks of chronic therapy or dose escalation; then every 3 months

Match the following characteristics below to their appropriate terms regarding Rheumatoid Arthritis: 1. ≥ 6 weeks of joint pain and/or stiffness (deformity not until later in disease progression) Fatigue, weakness, low-grade fever, loss of appetite 2. Tenderness, warmth, and swelling in affected joints (hands and feet); Symmetrical joint; nodules are possible

1. Symptoms 2. Signs

Bacterial-Polysaccharide vaccines contain long chains of sugar molecules that make up the surface capsule protein of the bacteria. 1. Pure Polysaccharide: 2. Conjugate Polysaccharide:

1. T-cell independent, which means that these vaccines can stimulate B cells without the assistance of helper T cells. . Children younger than 2 years are unable to form an immune response by this method. Example is Pneumococcal PPSV23 2. An added protein that changes the response to a T cell mediated response. Allows children younger than 2 years to form an immune response. Example is Pneumococcal PCV 13

For the following statements, state whether TRUE or FALSE: 1. NSAIDs and corticosteroids are preferred over colchicine for treatment of acute gouty attacks because they are highly effective with minimal toxicity. 2. For severe or polyarticular attacks, combination therapy with colchicine and NSAIDs can be considered.

1. TRUE 2. TRUE

INFLUENZA - NOMENCLATURE is shown below, What are the 3 stages that the virus goes through?

1. Transmission 2. Incubation 3. Viral Shedding

Treatment categories for MS include:

1. Treatment of exacerbations -Shorten duration and maybe decrease severity of attack 2. Disease-modifying therapies -Alter course of the illness -Diminish progressive disability over time 3. Symptomatic therapies -Important for maintaining QoL

T/F: 1. If pts develops persistent IFN antibodies, movement to a non-IFN is reasonable 2. If failing low-dose IFN, can increase dose, administer more frequently, or change to non-IFN (Could also add immunosuppressant agent, such as monthly methylprednisolone)

1. True 2. True

T/F: 1. Never give both PCV13 and PPSV23 at same visit 2. -f giving PCV13 at age 65, best to do PCV13, wait 12 months, then PPSV23

1. True 2. True -If PPSV23 was given first, then wait 12 months before giving PCV13

Match the following characteristics below for the following med: zanamavir (Relenza®) -- >/=7 same as adult Two inhalations BID within 48 hrs of symptoms; treat 5 days; first 2 doses separated by at least 2 hrs, then Q12H 1. Adult Prophylaxis: 2. Child Prophylaxis:

1. Two inhalations (10mg) daily within 36 hrs of contact for 7 days 2. 5 years or older; same dose as adults -Avoid in patients with respiratory disease -Avoid in patients with milk allergy

Match the following classifications of psoriasis with their BSA/PASI involvement. 1. Mild psoriasis 2. Moderate psoriasis 3. Severe psoriasis

1. ≤5% BSA involvement 2. PASI ≥8; BSA 5-10% 3. PASI ≥10; BSA ≥10% *mild (topical tx), moderate & severe (systemic tx)* *rule of 10s for severe* *Approach to therapy generally involves combination (can include both pharm and non-pharm)*

*KNOW* What is the dose of Colchicine (Colcrys)?

1.2 mg STAT followed by 0.6 mg 1 hr later

OTC --meltaway tabs, suppositories, suspension APAP has a max fever reduction seen around 2hrs. How is this medication dosed?

10-15 mg/kg/dose every 4-6 hours Only 1 concentration available max doses: Child: 75mg/kg/day; Neonate: 60 mg/kg/day; Adults: 4 g/day

Anakinra (Kineret) is considered to be used when you have failed everything else. What is its dosing?

100 mg SQ daily at same time each day -Can't combine with TNF inhibitors -Infections, malignancy, neutropenia, inject rx

The initial dose of allopurinol is?

100 mg daily renal adjustment dosing needed* Stage 4 CKD dose? 50 mg daily*

Allopurinol is titrated every 2-5 weeks to a max of 800 mg daily (based on uric acid level). What is the ideal maintenance dose?

100-300 mg/day

Dosing considerations of Rituximab (Rituxan) include what?

1000 mg IV infusion on days 1 and 15 with MTX -Administer subsequent courses every 24 weeks of based on clinical symptoms (No sooner than q 16 weeks) Infusion rx: premedicate with acetaminophen and an antihistamine prior to infusion

Dalfampridine (Ampyra) is the first agent to help improve walking in adults and takes up to 6 weeks to show efficacy. What are its dosage?

10mg po BID CI: in pts with history of seizures and moderate or severe renal impairment (CrCL<=50ml/min)

Upadacitinib (Rinvoq) can be used monotherapy or in combination with methotrexate or other nonbiologic DMARDs. How is this medication dosed?

15 mg PO once daily Do not use with other JAK, biologic, or potent immunosuppressant Do not start if absolute lymphocyte count <500/mm3, ANC <1,000/mm3, or hemoglobin <8 g/dL

For the following statements, state whether it applies to acute pain, chronic pain or BOTH: 1. Clinical presentation: a. Distress? b. May not have noticeable suffering? 2. Labs: a. No specific tests helpful? b. Imaging, Vit. D, TSH, B12 (neuropathic pain)? 3. Dependence/Tolerance: a. Unusual? b. Common? 4. Organic cause: a. Common? b. May not be present? 5. Environmental/ family issues: a. Small? b. Significant? 6. Treatment goal: a. Pain reduction? b. Pain reduction + Functionality?

1a. ACUTE 1b. CHRONIC 2a. ACUTE 2b. CHRONIC 3a. ACUTE 3b. CHRONIC 4a. ACUTE 4b. CHRONIC 5a. ACUTE 5b. CHRONIC 6a. ACUTE 6b. CHRONIC

KNOW SLE Treatment Approach of LUPUS INCLUDE:

1st Line: Antimalarials, NSAIDs 2nd Line: Corticosteroids 3rd Line: Immunomodulatory Agents

MMR is a live, SQ vaccine for measles, mumps, rubella. What are dosing concerns regarding this vaccine?

1st dose at 12-15 mos, 2nd dose at 4-6y/o

How should Herpes Zoster Shingrix (RZV) (IM) be given?

2 doses of RZV 2-6mos apart to adults 50 and older regardless of past episode of herpes zoster Unknown safety in preg. and immunocompromised

In patients with severe renal or hepatic insufficiency, repeat treatment for acute attacks with colchicine should be no more often than every?

2 weeks

Diclofenac Topical solution (Pennsaid)

2% - Use two pumps of the solution (40 mg) into the palm of the hand ◦Apply evenly to front, back, & sides of each affected knee ◦Repeat twice daily

Dose dependent deaths:

20% overdoses with MME <100mg/day 80% with MME >100 mg/day Increased risk of overdose -Higher MME (>50 mg/d) -Benzodiazepines -Sedative/hypnotics -Psychiatric disorders -Substance abuse history -Illicit substances -Male gender -Increased age -Number of prescriptions

Sarilumab (Kevzara) Monotherapy or combo with nonbiologic DMARDs; Do not initiate if ANC is <2,000/mm3, platelets are <150,000/mm3 or if ALT or AST are >1.5 times ULN. How is this medication dosed?

200 mg SQ q 2 weeks If side effects occur, reduce to 150 mg Monitor: neutrophils/platelets/lipid panel 4-8wks

KNOW First-degree relatives with SLE are ______ more likely to develop SLE than the general population

20x Concordance rate is 24-69% for identical twins 2-5% for fraternal twins or other siblings

Interferon-β1b reduces annual relapse rate and MRI burden of disease, restore suppressor function, and alter cytokine and matrix metalloproteinase expression. What is the dosage of this medication?

250 mcg SC every other day -Betaseron and Extavia First agent proven to alter natural course of MS -Have antiviral and anti-proliferative effects -Reduce antigen presentation and T-cell proliferation

Baricitinib (Olumiant) can be used monotherapy or in combination with methotrexate or other nonbiologic DMARDs. How is this medication dosed?

2mg PO daily BBW: -Serious infections -Malignancies -Thrombosis

Duration of therapy of Corticosteroids [prednisone, methylprednisolone, prednisolone, dexamethasone] in the treatment of asthma exacerbations:

3 to 10 days (~7 days on average) Used in addition to SABA for all mod-severe exacerbations Greatest advantage within 1 hour of initial presentation AE: Hyperglycemia, hypertension, acute psychosis wean is not necessary w/ this short term course

Glaucoma is progressive irreversible optic nerve damage. The impact serves as what?

3% of everyone above age 40 worldwide -1 in 40 adults over age 40 Of those untreated or failing to attain targeted IOP reduction, 80% have continued visual field loss -Progression to blindness in treated patients is 4-22%

CASE: -Patient is on hydromorphone IV 12mg/day (total daily dose) -Want to convert to morphine ER (PO) -Hospital policy - reduce dose by 50% when switching opioids What is your final morphine ER regimen?

30 mg PO morphine = X mg PO morphine 1.5 mg IV hydromorphone 12 mg IV hydromorphone 1.5 mg IV hydro * X = 30 mg PO morph * 12mg IV hydro X = 240mg PO morphine Reduce dose by 50%: 120 mg PO morphine New regimen: Morphine ER 60mg PO BID

Duloxetine (Cymbalta®) May be used in combo w/ NSAIDs /APAP [inc. bleed risk]. What are dosing considerations pertaining this medication?

30 mg once daily for one week, then 60 mg once daily -Label has a max dose of 60 mg per day, but there is some evidence for additional benefit up to 120 mg daily with potential increase in adverse effects. Taper over 2-4 weeks to discontinue Avoid in CrCl <30 mL/min AE - nausea, dry mouth, fatigue, somnolence

Non-Pharmacologic Treatment Options for FEVER Fever causes the child to lose more fluids and burn more calories. Can easily become dehydrated. Increase fluid intake by:

30-60 mLs per hour (sports drinks, fruit juice, water, electrolyte replacement products or ice pops) •Maintain adequate hydration •Sponging or cool baths with >104˚F (40◦C) -Ice baths are dangerous and not recommended -Isopropyl alcohol baths are not recommended •Lightweight clothing •Remove blankets •Maintain comfortable room temperature

Natalizumab (Tysabri) is approved for use in relapsing pts w/ inadequate response or intolerance to other therapies. What are its dosage concerns?

300mg IV q 4 weeks AE: PML risk, Depression, Fatigue, Respiratory infection, Arthralgia Pts must be screened at baseline and every 6 months with a JCV test while on treatment

Scenario A patient is taking oxycodone ER 30mg PO BID, and hydrocodone/APAP 5mg/325mg PO Q6H PRN. She tells you she takes 2-4 hydrocodone/APAP tabs per day, and takes her oxycodone as directed. What is the total daily MME if the patient takes the maximum prescribed dose?

30mg oxycodone * 2 tab/day = 60 mg/day oxycodone 60 mg oxycodone = 20 mg oxycodone X mg morphine 30 mg morphine X = 90 mg MME from oxycodone hydrocodone 5mg * 4 tabs = 20 mg/day hydrocodone Note, hydrocodone 30mg = morphine 30mg: 1:1 ratio 20 mg hydrocodone = 20 mg MME from hydrocodone Total MME if patient is taking the maximum prescribed dose = 110 mg MME per day

Adalimumab (Humira) is a Anti-TNFα monoclonal antibody (IgG1) that binds to TNF-alpha, interfering with binding to TNFα receptors that is indicated for adults with moderate to severe active RA AND Psoriasis & Psoriatic Arthritis. Juvenile idiopathic arthritis, Crohn's disease, Ulcerative colitis, & Ankylosing spondylitis. What is the dosing of this medication?

40 mg SQ every other week can continue methotrexate, other nonbiologic DMARDS, corticosteroids, NSAIDs, and/or analgesics patients not taking concomitant methotrexate may increase dose to 40 mg every week

Certolizumab pegol (Cimzia) is a pegylated humanized antibody fragment to TNF-alpha and indicated for moderate to severe active RA, Plaque psoriasis, Psoriatic Arthritis, Crohn's, & Ankylosing spondylitis. What is the dose of this medication?

400mg SQ at Week 0, 2, 4, then 200mg every other week -Alternative maintenance dose: 400 mg SQ every 4 weeks -Alone or with MTX AEs similar to TNF Profile: Infections 38%; URI 6-20%

BBW: Risk of QT prolongation; do not exceed 20mcg/hr patch Buphrenorphine transdermal (Butrans):

5 mcg/hr, 7.5 mcg/hr, 10 mcg/hr, 15 mcg/hr, 20 mcg/hr -Changed weekly -Applied to outer arm, upper chest, side of chest, upper back; do not use same site for at least 3 weeks ▪Do not expose to heat!

Intraarticular - Hyaluronic Acid is expensive ($1K per course). When is the max effect achieved from this medication?

5-13 weeks and lasting 24 weeks AE: pain, swelling, arthralgia for few days strongly against use in hip and conditionally against use in knee or first CMC

Golimumab (Simponi) is a Human monoclonal antibody to TNF-alpha that is indicated for moderate to severe active RA with methotrexate, Psoriatic Arthritis, UC, Ankylosing spondylitis. What is the dose of this medication?

50 mg SQ once monthly with methotrexate IV: 2 mg/kg at weeks 0, 4, and then every 8 weeks thereafter with methotrexate AEs similar to TNF profile

Etanercept (Enbrel) binds tumor necrosis factor (TNF) and blocks its interaction with cell surface receptors and is a recombinant DNA-derived protein composed of tumor necrosis factor receptor (TNFR) linked to the Fc portion of human IgG1. what is the dose of this medication?

50 mg SQ once weekly (off-label 25 MG BIW) -Typically self injection -may continue methotrexate, glucocorticoids, salicylates, NSAIDs, or analgesics during etanercept therapy

*KNOW* What is the dose of Indomethacin?

50 mg TID until pain tolerable

Dose Adjustment Example Patient is on Oxycontin (oxycodone ER) 20mg PO BID with oxycodone 5mg PO Q6H PRN pain. The patient is still experiencing severe pain despite taking oxycodone at the maximum prescribed dose. How would you adjust this regimen?

5mg * 4 doses = 20 mg oxycodone (IR) Guidance: Increase long-acting regimen by ~25-100% of total daily short-acting dose 25-100% of 20 mg: 5-20mg Increase Oxycontin by a total of 5-20mg daily (currently 40mg daily)

Fentanyl patches are for patients who have been using what?

60mg MME daily for at least 7 days Dose should not be titrated more frequently than every 3 days initially, and 6 days thereafter!

Suicide rate as high as _____ times that of the general population has been reported

7 -Does not directly decrease life expectancy -Secondary complications -Decrease in life span seen most in pts with rapidly progressive disease

Teriflunomide (Aubagio) has a dosage form of what?

7 or 14mg po once daily Black-box warnings -Hepatotoxicity - LFTs (within 6 mths of starting, and monthly for 6 months) -Teratogenicity - pts should be placed on birth control

*KNOW* What is the starting dose of methotrexate?

7.5-15mg qweek can divide into 3 doses given 12 hours apart still weekly*

Most common STD w/ high risk types found in 99% cervical cancers, Human Papillomavirus (HPV) is aluminum adjuvanated (helps create a stronger immune response in the patient's body.), given IM and recommended for whom?

<26 y/o: 27-45 clinical decision making 3-dose series: 0, 1-2 months, 6 months 2-dose series: 0, 6-12 months VLP: the outside shell looks like the virus leading to a bigger immune response. HPV 9 adds protection against five additional HPV types—31, 33, 45, 52 and 58— which cause approximately 20 percent of cervical cancers and are not covered by previously FDA- approved HPV vaccines.

Acetaminophen: IV Ofirmev is an expensive medication but may reduce use of other agents (opioids). What are the considerations with dosing?

<50 kg: 15 mg/kg every 6 hours or 12.5 mg/kg every 4 hours ≥50 kg: 650 mg every 4 hours or 1000 mg every 6 hours

Systemic influenza symptoms:

<5y/o: Fever, Vomiting, Diarrhea Adults: Rapid symptom onset Fever, usually >100F Chills, Sweats, HA, Myalgia, Malaise, Substernal soreness, Photophobia Elderly: Fever, >99F, Lassitude, Confusion

Respiratory influenza symptoms:

<5y/o: Rhinitis Pharygnitis Adults: Nonproductive cough, sore throat Elderly: Nasal obstruction

What is the goal uric acid level for gout prophylaxis?

<6 mg/dL

ULT can be discontinued if levels are what?

<7 mg/dL x 5 years with no attacks.

OTC Naproxen pts may take 400mg as the first dose. This medication is approved for whom at what dose?

>12 years old 200 mg every 8-12 hours Max 600 mg per day

OTC --suspension, concentrated drops, chewable tablets Ibuprofen (IBU) maximum fever reduction seen around 2 hours. This medication can be used in whom?

>6 months old 5-10 mg/kg/dose every 6-8 hours max single dose: 400 mg; MDD: 40 mg/kg/day Two concentrations available

Hyperuricemia is elevated serum urate level of what?

>7 mg/dL

rheumatoid arthritis (RA)

A chronic, progressive autoimmune disease causing inflammation in the joints and other body systems. Which results in painful deformity and immobility. Most commonly seen in the fingers, wrists, feet, and ankles. Affects more than just joints. Can affect skin, eyes, lungs, heart and blood vessels

*QUESTION* AJ has been on allopurinol 300mg daily for 6 months, but his uric acid level continues to remain high at 7.8 mg/dL. What could be appropriate therapy to consider adding to AJ's regimen? (Select all that apply) A. Probenecid B. Fenofibrate C. Febuxostat D. Anakinra E. Lenisurad F. Losartan

A, B, E, F *Do NOT USE both xanthine oxidase inhibitors at the same time* *Anakinra is only considered in severe acute refractory attacks*

*KNOW* Gout prophylaxis is indicated if:

A. 2 or more attacks per year B. Multiple tophi C. History of urolithiasis (stones in the urinary tract) D. CKD stage 2+ (GFR/CrCL <89) E. Uric acid level >10 mg/dL *NOTE: Patient does not have to meet all the criteria for prophylaxis; 1 is enough to warrant prophylaxis*

Which of the following is TRUE regarding Febuxostat (Uloric)?

A. AE's include mild ↑LFTs, nausea, abdominal pain, diarrhea, arthralgias B. It has shown evidence of CV harm C. It does not require dosage adjustment in renal or hepatic impairment D. Its dosed at 40 mg daily *allopurinol requires dose adjustment BUT febuxostat does not*

*KNOW* Infusion reactions due to Infliximab (Remicade) can be minimized via pre-medication, slow rate of infusion or administration with MTX or other immunosuppressants. Agents used for pre-medication include:

A. APAP & Diphenhydramine 90 minutes prior to infusion B. Corticosteroids

Methotrexate (immunosuppressant) is indicated for moderate to severe plaque psoriasis & psoriatic arthritis. It is contraindicated in:

A. Active infection B. Renal impairement C. Leukopenia/thrombocytopenia D. Alcohol abuse E. Pregnant & breastfeeding (teratogenic!) *slightly less effective than cyclosporine*

NSAIDs should be avoided in:

A. Active peptic ulcer disease B. Active bleeding C. Severe renal impairment D. Uncompensated congestive HF E. Hypersensitivity to aspirin or NSAIDs concomitant use of aspirin as antiplatelet therapy & NSAIDs for gout should be cautioned*

Psoriasis involves the:

A. Adaptive immunity system B. Innate immunity system

First line prophylaxis (ULTs) agents for gout include:

A. Allopurinol (Zyloprim) B. Febuxostat (Uloric)

NSAIDs interact with (DDIs):

A. Antihypertensives -- dec. antihypertense effect B. Anticoagulants/Antiplatelets -- inc. bleed risk C. Bisphosphonates (alendronate) -- inc. GI irritation D. Salicylates -- dec. NSAID effect E. Methotrexate, phenytoin & lithium

Which of the following is TRUE regarding gout prophylaxis?

A. Any change in uric acid levels (up or down) can precipitate attack B. Anti-inflammatory agent should be administered during initiation to prevent acute flare C. ULT can be started during an acute attack D. Low dose colchicine is 1st line for anti-inflammatory prophylaxis NSAIDs are 1st line short term; Colchicine is 1st line long term*

Guselkumab (Tremfya):

A. Approved for moderate to severe plaque psoriasis B. Selective inhibitor of IL-23 C. SE's are generally mild D. Patient should be screened for TB

Classification of psoriasis is based on?

A. BSA B. Psoriasis Area & Severity index measurements (PASI) PASI looks at 4 areas of the body, and combines the amount of area involved (0-6) and the severity (0-4) of the lesions*

NSAID use should be cautioned in patients with:

A. CHF B. Liver disease with ascities C. Compromised renal function D. Concomitant diuretic use

Which of the following screenings are recommended in patients with psoriasis?

A. CV risk B. Metabolic syndrome C. Obesity

Which of the following combinations are appropriate for the management of a severe/polyarticular acute gout attack?

A. Colchicine + NSAIDs B. Colchicine + CCS C. IA + others ongoing ULT (Urate-Lowering Therapy) should not be interrupted during an acute gout attack*

Signs and symptoms of GI bleeding include:

A. Coughing up or vomiting blood or something that looks like coffee grounds B. Having black or tarry stools C. Having abnormal stomach pain or burning D. Unexplained weight changes

Keratolytics:

A. Decrease hyperkeratosis (remove scales, smooth skin) B. Agents include salicylic acid, urea & glycolic acid (applied 2-3x daily) C. Salicylic acid improves penetration of corticosteroids D. Salicylic acid can be associated with salicylism (salicylate poisoning) *salicylism is characterized by ringing in the ears, nausea & vomiting*

Interaction between ________ result in epidermal hyperplasia & dermal inflammation (psoriasis).

A. Dermal dendritic cells B. Activated T-cells C. Cytokines (TNF-a, interferon gamma, IL-1) D. Growth factors

Patients with increased likelihood of septic arthritis include:

A. Diabetics B. Alcohol users C. Elderly

Secondary cause Which of the following drugs/conditions can decrease renal clearance of uric acid resulting in uric acid reabsorption (under-excretion)?

A. Diuretics B. Salicylates (<2g/day) C. Nicotinic acid (Vitamin B3) D. Dehydration E. Ethanol Others: TB meds (PZA, ethambutol), cyclosporine*

All side effects of TCS increase based on:

A. Drug potency B. Duration of use C. Size of area treated

Which of the following is TRUE regarding prophylaxis with Xanthine oxidase inhibitors?

A. Drugs include allopurinol & febuxostat B. They impair xanthine oxidase C. They are 1st line & are effective in overproducers & underexcreters D. Their effectiveness is dose-dependent

Renal SE's of NSAIDs include:

A. Dysfunction caused by prostaglandin inhibition B. Increased BP (average 5mmHg increase) Others: CNS, Hepatotoxicity (rare), Respiratory (asthma exacerbation), Dermatologic monitor renal function with creatinine & BUN (blood tests)*

Non-pharmacological therapy options of gout include:

A. Education on adherence & monitoring B. Weight loss C. Diet focused on vegetables, cherries & low-fat dairy D. Elimination of purine rich foods such as organ meats & shellfish E. Limiting or eliminating alcohol & high fructose corn syrup (<2 drinks/day men & <1 drink/day women) F. Medication evaluation to rule out drugs that may elevate uric acid

MOST common areas for plaque psoriasis is?

A. Elbows B. Knees *but can develop anywhere on the skin*

*KNOW* Which of the following TNF inhibitors have the highest quality of evidence in psoriasis?

A. Etanercept (Enbrel) B. Adalimumab (Humira) C. Infliximab (Remicade)

Disadvantages of biologics/BRMs?

A. Expensive B. Increase risk of infection insurance companies require failure of previous therapies before use of biologics*

High potency corticosteroids are very effective but are associated with increased ADRs & should be avoided on the:

A. Face B. Folds of skin C. Groin

Osteoarthritis:

A. Failure of the movable, synovial-line joint B. Is degenerative & inflammatory C. Is the common cause of disability resulting in pain & altered function of joints D. Is highly variable

Adverse effects of NSAIDs include:

A. GI B. Kidney C. CV D. CNS CV effects: HTN & sodium/fluid retention*

Allopurinol is associated with severe "Allopurinol Hypersensitivity Syndrome". Risk is increased if:

A. HLA-B*5801 screening B. Asian, East Indian, African American C. Renal impairment/insufficiency D. Thiazide diuretic use rash = stop the drug* headache, diarrhea = don't stop drug unless its severe; treat symptoms*

Which of the following should you have before beginning biologic therapy?

A. History & physical B. Updated med list C. CBC with platelets D. LFTs E. Hepatitis panel F. PPD G. Vaccines (pneumococcal, hep A & B, Influenza, Td)

Gout can be classified as:

A. Hyperuricemia B. Recurrent attacks of acute gouty arthritis C. Tophi (long-term gout) acute: deposits in synovial fluid) tophi/chronic: deposits in joint tissues)

Ustekinumab (Stelara) *SubQ*:

A. IL-12 & IL-23 antagonist B. Its used for moderate to severe chronic psoriasis, psoriatic arthritis & crohn's disease C. Its effective after just 2 doses D. Watch for malignancy and infection E. Baseline CBC, then monthly for 3 months, then periodically

Interleukin-1 inhibitors (Anakinra & canakinumab) are ONLY considered for severe acute refractory attacks. They are NOT frequently used due to:

A. Immune hypersensitivity reactions B. Injection-site reactions C. Neutropenia's D. Risk of infections E. Malignancies

Secondary causes of uric acid OVERPRODUCTION include:

A. Increased breakdown of tissue nucleic acids B. Excessive rates of cell turnover C. Tumor lysis syndrome due to use of cytotoxic medications

NSAIDs are the mainstay therapy for acute attacks due to excellent efficacy & minimal toxicity when used short term. Which of the following NSAIDs are FDA approved to treat acute gout attacks?

A. Indomethacin B. Naproxen C. Sulindac

Gout diagnosis triad includes:

A. Inflammatory monoarthritis B. Elevated serum uric acid levels C. Response to colchicine

Which of the following is TRUE regarding Fenofibrate for gout prophylaxis?

A. It has a sustained reduction in serum urate (20-30%) B. Initiation does NOT cause acute flare

Which of the following is TRUE regarding Losartan for gout prophylaxis?

A. It inhibits renal absorption of uric acid B. It increases urinary excretion of uric acid C. It decreases risk for stone formation D. It alkalizes the urine

Which of the following is TRUE regarding psoriatic arthritis?

A. It presents with arthritis-like symptoms B. It develops in up to 30% of psoriasis patients C. It usually develops 10 years after skin diagnosis D. Often affects distal interphalangeal joints Also: Knees, elbows, wrists, ankles

Which of the following is TRUE regarding osteoarthritis & its presentation?

A. Its 2nd to CV disease in causing severe chronic disability B. Its present in 50% of people over 65 y/o & present in almost 100% of people over 75 y/o C. Older women are 2x as likely as men to have OA of the knee & hands D. Women are more prone to inflammatory OA of proximal & distal joints of hands

Which of the following is TRUE regarding gout?

A. Its 3x more common in men than women B. It peaks at age 50+ C. Development before age 30 indicates enzyme defect or renal disease D. It was considered the "disease of kings"

Which of the following is TRUE regarding Colchicine (Colcrys)?

A. Its a highly effective antimitotic agent B. It should be initiated within the first 36 hrs of attack C. Its usually reserved for patients with CI's, intolerance or unresponsiveness to NSAIDs D. Its equally effective as the old traditional dosing (with less GI toxicity) 77% diarrhea with old & 23% diarrhea with new* Its also very effective in first 24hrs*

Which of the following is TRUE regarding "classic acute gout"?

A. Its a rapid, localized onset of excruciating pain, swelling & inflammation B. It lasts 3-14 days C. It has spontaneous resolution D. Often monoarticular at first (great toe, insteps, ankles, heels, knees, wrists, fingers, elbows)

Which of the following is TRUE regarding tazarotene (tazorac)?

A. Its a topical retinoid B. Its applied at bedtime C. Can be used in combination with CCS to ↓ ADRs and ↑ efficacy D. Can be combined with phototherapy E. Should NOT be applied to >20% of BSA

Which of the following is TRUE regarding probenecid for gout prophylaxis?

A. Its a uricosuric (increases renal clearance of uric acid) B. AE's include urolithiasis, precipitation of acute gouty arthritis, GI irritation, rash & hypersensitivity C. Its used in underexcreters urolithiasis (most common) can be prevented by urine alkalization & maintaining urine flow*

Which of the following is TRUE regarding calciptriene/calcipotriol (dovonex)?

A. Its a vitamin D analog & the only one on market in the US B. Its indicated for mild psoriasis C. Well tolerated & is the safest long term topical treatment D. Mild adverse effects that are alleviated with continued use E. Its inactivated by ultraviolet A (UVA) light after UVA light exposure F. Pregnancy class C *systemic AEs are rare* Dosing: Applied 1-2 times daily, Strength: 0.005%

Which of the following is TRUE regarding Lesinurad (Zurampic)?

A. Its dosed at 200 mg daily & MUST be given in combination w/ allopurinol (at least 300 mg/day) or febuxostat B. It has a BBW for acute renal failure (monotherapy) C. It should NOT be initiated in patients with CrCl <45 mL/min D. It showed no difference in tophi resolution compared to allopurinol (gout flare in 12 months)

*KNOW* Which of the following is TRUE regarding Acitretin (Soriatane) - oral retinoid?

A. Its taken with food B. Its dosed at 25-50 mg daily C. Its teratogenic & decreases effectiveness of birth control D. Cannot become pregnant for 2 years post therapy E. Cannot donate blood for 1 year post therapy Works better if combined with phototherapy or topical calcipotriol*

Joints MOST commonly affected in OA include:

A. Knees B. Hips C. DIP (Heberden's nodes) D. PIP (Bouchard's nodes) E. First carpometacarpal F. Cervical & lumbar spine G. First MTP joints (big toe)

Which of the following is TRUE regarding capsaicin?

A. Major ingredient in chili peppers B. It has no action on capillaries, so no redness C. Treats pain including neuropathic (but not inflammation of OA and RA) D. Rx product is Qutenza (PHN pain) & OTC products are Capzasin, Icy Hot Arthritis, Zostrix (0.025% and 0.075%) Recommended for knee OA

Based on potential agents for treating, which of the following labs should be considered "pretreatment?

A. Metabolic profile B. CBC C. LFTs D. Hepatitis & TB evaluation

Prophylaxis is NOT indicated in:

A. Mild first episode of gout & responded to treatment B. Serum urate levels minimally elevated

Emollients (moisturizers) apply 3-4 times daily:

A. Minimize dryness & irritation B. They are used as adjunct to treatment (maintenance) C. They are safe, inexpensive & reduce scaling/itching D. May leave skin greasy E. Can be formulated as lotions, creams, ointments AE: Folliculitis; Allergic or irritant dermatitis

Goals of therapy for psoriasis include:

A. Minimize or eliminate signs of psoriasis, plaques, scales & irritated skin B. Relieve itching & heal excoriations C. Reduce the frequency & risk of flare-ups D. Address comorbid conditions such as psoriatic arthritis, hypertension, dyslipidemia, diabetes, or clinical depression E. Manage triggers and/or predisposing risk factors F. Avoiding or minimizing ADRs G. Maintaining or improving the patient's quality of life

Which of the following is TRUE regarding anthralin (psoriatec)?

A. Must apply ONLY to affected areas B. Use gloves C. Applied to plaque lesions for 2 hours or less and then wipe off D. Zinc oxide ointment or a non-medicated stiff paste should be applied to the surrounding normal skin to protect it from irritation

First line therapy options for acute gout attacks include:

A. NSAIDs B. Corticosteroids C. Colchicine

Colchicine is associated with dose-dependent GI effects. These include:

A. Nausea B. Vomiting C. Diarrhea D. Cramps E. Abdominal pain

Principles of OA drug therapy include:

A. Non-pharm therapy as 1st line B. Medications do NOT alter disease course but are used to treat symptoms C. Self-care is appropriate if diagnosed

Advantages of using low potency corticosteroids include: *disadvantage?*

A. OK to apply almost everywhere B. Less ADRs C. Less cost *disadvantage? less effective*

High serum uric acid (gout) is associated with:

A. Obesity B. Alcohol intake C. Diet high in meat, seafood, sugar D. Increased SCr, BUN & BP

Arthritis is more common in:

A. Older age B. Rural areas C. Smokers D. Lower education E. Less physically active F. Higher BMI

Capsaicin administration:

A. Onset of relief takes ~14 days may take 4-6wks B. Should be used consistently, 3-4 times per day C. Use gloves, wash hands, avoid touching sensitive areas (for hand leave 30min. before washing)

Pain in osteoarthritis can result from:

A. Osteophyte growth with stretching of the periosteum B. Microfractures C. Synovitis D. Damage to ligaments & meniscus

Corticosteroids should NOT be used long term. Long term use may cause:

A. Osteoporosis B. HPA suppression (hypothalamic-pituitary axis) C. Cataracts D. Muscle deconditioning

Probenecid is contraindicated in?

A. Overproducers B. History of urolithiasis C. Renal impairment (CrCL <50 mL/min)

Clinical findings of OA include:

A. Pain (deep ache, localized to the joint) B. Pain aggravated by joint use & relieved by rest C. No morning stiffness or less than 30 minutes D. Nocturnal pain & additional stiffness in advanced OA E. Joint motion limitation F. Crepitus (grating or crackling with motion)

Apremilast (Otezla):

A. Phosphodiesterase-4 inhibitor B. Used for moderate to severe plaque psoriasis & psoriatic arthritis C. Titrated over 5 days to maintain dose of 30 mg BID on day 6 D. Associated with significant clearing of plaques E. Monitor depression, weight, DDIs & CrCL

Second line prophylaxis (Uricosurics) agents for gout include:

A. Probenecid B. Losartan C. Fenofibrate *probenecid is 1st line if xanthine oxidase inhibitors are contraindicated* *guidelines do mention Lenisurad*

Adverse effects of Entaracept (Enbrel) include:

A. Pruritic injection site reactions B. Headaches C. Infection D. URTI & LRTIs

Acetaminophen:

A. Recommended for Knee, hip, hand B. 325mg-1000mg q4-6h (MAX 4g/day FDA rec. OTC MAX: 3g/day) C. Lacks GI toxicity (major advantage) D. Concern for hepatotoxicity with long-term use above 3 g/day Study show only a modest benefit

Goals of OA therapy include:

A. Relieving pain & stiffness B. Maintaining or improving joint mobility C. Limiting functional impairment D. Maintaining or improving quality of life E. Avoiding or minimizing medication adverse effects

IL-17 antagonists are associated with risk of exacerbation of or new onset irritable bowel disease & hypersensitivity reactions. Agents include:

A. Secukinumab (Cosentyx) B. Ixekizumab (Taltz) C. Brodalumab (Siliq)

Coal tar is well tolerated & inexpensive but:

A. Smells bad B. Can stain skin & clothes C. Is NOT recommended on acutely inflamed skin D. May not be efficacious *2nd line agent; not well studied*

Colchicine has DDIs with:

A. Statins, fibrates, chemotherapeutics B. PgP inhibitors C. CYP3A4 inhibitors such as macrolides, verapamil, azoles, grapefruit juice *PgP inhibitors & CYP3A4 inhibitors require dosage adjustments when used with colchicine*

ADRs of topical steroids include:

A. Striae B. Skin atrophy C. Hyperproliferation of skin D. Telangiectasia

*KNOW* Methotrexate:

A. Supplement with folic acid 1mg daily B. Long term use associated with hepatotoxicity C. CBC with differential & platelets obtained at baseline, 7-14 days after initiating therapy or dosage increase, every 2-4 weeks for the 1st few months then every 1-3 months depending on leukocyte count & stability of patient D. Should be D/C 3 months before conception

Which of the following is TRUE regarding Tramadol (Ultram)?

A. Synthetic opioid agonist, weakly inhibits reuptake of norepinephrine and serotonin B. IR: 25-50 mg Q 6 hr (Max of 400 mg/day, 300 mg/day in elderly) C. Similar effect to ibuprofen for hip and knee OA D. Can be combined with NSAID or APAP (Ultracet) E. Taper for D/C F. CrCl <30 mL/min, decrease dose of IR and avoid ER

Overall treatment goals of gout are:

A. Terminate the acute attack B. Prevent recurrent attacks C. Prevent complications of chronic urate crystals in tissues D. Prevent/reverse comorbidities such as obesity, elevated triglycerides & hypertension

Which of the following is TRUE regarding NSAIDs?

A. They are analgesics at lower doses & anti-inflammatory at higher doses B. All work equally well overall with patient-patient variability C. MOA is based on blockade of prostaglandin synthesis through inhibition of COX-1 & COX-2 D. First line treatment with consideration of safety (Avoid in age >75 years; assess GI and CV risk)

Which of the following are appropriate photochemotherapy combos with topical agents used in psoriasis?

A. UVB + crude coal tar (Goeckerman regimen) B. UVB + anthralin (Ingram regimen) for enhanced efficacy

What are ways we could enhance topical CCS potency?

A. Use of a more occlusive vehicle (ointment) B. Addition of a penetration enhancing agent (ex: urea, salicylic acid, propylene glycol) C. Addition of a fluorine atom D. Addition of valerate, dipropinate or acetonide group

Calcineurin (protopic) inhibitors:

A. Used off-label for mild-moderate psoriasis & inverse psoriasis B. Pimecrolimus has better skin penetration than tacrolimus C. Should not be used in patients with weak or compromised immune systems D. BBW for rare cases of lymphoma & skin malignancy (use sunscreen with SPF 30+)

Septic arthritis is characterized by:

A. White cells & purulence from synovial fluid in affected joints B. High fever C. Elevated WBC count D. Multiple affected joints

AOM: Antibiotic Therapy

AAP: for no improvement for PCN allergy rec clindamycin + 3rd gen ceph (JHACH clinda S.pneum only ~80% S + oral ceph ~70-80% therefore more than likely not offering better coverage than the "failed" original regimen and certainly less superior 3rd gen ceph: parenteral: Cefotaxime, Ceftazidime, Ceftriaxone oral: Cefixime, Cefdini

Match the following description to either acute or chronic gouty nephropathy. -Precipitation of uric acid crystals in collecting ducts/ureters -Acute renal failure resulting in total blockage of urine flow

Acute gouty nephropathy

CNCP Opioid Recommendations:

Acute pain (≤ 4 weeks) ▪Moderate to severe pain that is having impact on function or quality of life ▪Crush injuries, burns, fractures, significant tissue damage, spinal cord injury, phantom limb pain, etc. Post-operative Chronic pain (>3 months) ▪Consider if other evidence-based approaches for functional restoration have failed

What medications also treat Plaque Psoriasis & Psoriatic Arthritis. Juvenile idiopathic arthritis, Crohn's disease, Ulcerative colitis, & Ankylosing spondylitis?

Adalimumab (Humira) Infliximab (Remicade) Golimumab (Simponi) Certolizumab pegol (Cimzia)

*KNOW* _________________ loses its efficacy if stopped & restarted.

Adalimumab (Humira): 80 mg SQ first week, then 40 mg SQ the next week, then every 2 weeks thereafter

NAS Pharmacologic Treatment: Phenobarbital Prolonged half-life and concerns with neurodevelopmental effects. When would we use phenobarbital?

Adjunctive therapy for polysubstance exposure or non-opioid withdrawal -Not as effective as an opiate for monotherapy of opioid withdrawal -Does not improve GI symptoms

Acetaminophen (Tylenol) Per FDA, max APAP per dosing unit (combo products) = 325 mg! Watch for Liver toxicity, avoid with alcohol and stop taking if severe skin rash. How is this medication dosed?

Adults: 325-1,000mg PO Q4-6H -Max daily dose = 4,000 mg/day ▪~3,000 mg/day OTC unless directed by a healthcare provider Pregnancy Category C ▪Preferred agent for mild pain during pregnancy ▪(note: NSAIDS are contraindicated!!!) Children: 10-15 mg/kg PO Q4-6H

Aspirin should be avoided w/ NSAID hypersensitivity (nasal plyps, asthma) and for cardio protection is used at 81mg. The dosage for analgesic effect is what?

Adults: 325-1000mg PO Q4-6H (max: 4g/day) found in excedrin ↑BP, renal impairment, severe skin rash Salicylate overdose: tinnitus

_____________ patients respond less well to IV cyclophosphamide than whites or Asians

African American and Hispanic Patients of African or Hispanic origin tend to respond better to mycophenolate than cyclophosphamide Asian patients require lower doses of mycophenolate

Conditions that increase the risk of influenza complications:

Age -Adults >50 years -Children 6 to 59 months old Immunosuppression, including HIV infection Pregnancy during influenza season Chronic medical conditions -Pulmonary (emphysema, asthma), cardiovascular, metabolic (diabetes), renal dysfunction, hemoglobinopathy Residents of long-term care facilities Compromised respiratory function or aspiration risk

The following dosing characteristics below refers to which medication? IV formulation 1. 1st course: 12mg IV (over 4 hours) daily for 5 days 2. 2nd course: 12mg IV daily for 3 days (12mos after 1st course)

Alemtuzumab (Lemtrada) Pre-medicate with 1g methylprednisolone (or equivalent) immediately prior to infusion and for the 1st 3 days Administer antiviral agents for herpetic prophylaxis on 1st day of dosing and for minimum of 2 mths after completion or CD4 count is > 200 REMS program

MENINGOCOCCAL A, C, W, Y IM Should be given to whom?

All adolescents ►Give 1 dose of MCV4 at age 11-12 years ►Give 1 booster dose of MCV4 at age 16 years ►Give only 1 dose if first dose after age 16 years Not routinely recommended after age 19 years Adults with certain medical conditions (i.e., persistent complement component deficiency, anatomical or functional asplenia, HIV) Give 2doses of MCV4 at least 8weeks apart

Drug Causes: Adrenergic agents

Alpha agonists - phenylephrine, epinephrine, ephedrine, naphazoline Beta2 agonists - may occur from use of nebulizer Indirect sympathomimetics - amphetamines, MAOIs, cocaine Occurs because of mydriasis

Fatigue (One of most common complaints) Can be severely disabling, typically presents in mid to late afternoon. Can increase with heat exposure, exertion, current infection, spasticity, weakness, and depression. What meds. are involved in this symptom?

Amantadine (100mg po bid) is used often and may offer significant relief -ADHD medications (methylphenidate, etc) are also used -Modafinil (Provigil) and armodafinil (Nuvigil) also used

Uncomplicated CAP 1ST LINE OUTPATIENT: Previously healthy, appropriately immunized

Amoxicillin 90 mg/kg/DAY divided BID - TID* •TID regimen - Why?: To achieve appropriate amoxicillin exposure in lung even with relatively resistant pneumococci

Uncomplicated CAP ALTERNATIVES OUTPATIENT:

Amoxicillin/clavulanate 90 mg/kg/DAY divided BID - TID •Consider if patient received Amoxicillin within 30 days •Consider if patient not vaccinated against H.influenzae type b •Consider for non-severe penicillin allergy •Oral cephalosporin that has substantial activity against S.pneumoniae: cefpodoxime, cefprozil, or cefuroxime doses studied in children are inferior to high-dose amoxicillin •NOT cefdinir

Inpatient Empiric Therapy First Line: Inpatient (non-ICU): previously healthy, appropriately immunized

Ampicillin 50 mg/kg/dose IV q6h (max dose: 2000 mg) •Penicillin G is also appropriate first-line therapy

NSAIDs for the use of RA is a great option for adjunct therapy however it is not used for monotherapy despite anti-inflammatory effects because it does not alter the progression of diseases. What purpose does do they serve?

Analgesia and anti-inflammatory -Inhibit PG synthesis not enough -Reduces pain and stiffness

•More common in Inuit, Chinese, and Asian-India •30% of glaucoma world-wide •50% of blindness world-wide The following characteristics above describe what?

Angle Closure Glaucoma Severe ocular pain, blurred vision, halos around lights, headache, nausea, vomiting

Clinical presentation for RDS (respiratory distress syndrome) What is used for PREVENTION?

Antenatal corticosteroids -Betamethasone 12 mg IM Q24h x 2 doses is administered to women in preterm labor Promotes fetal lung maturation and surfactant production Reduces mortality and incidence

Historically used only in those with skin and joint manifestations

Antimalarial Agents (Hydroxychloroquine > Chloroquine) Reduces concentrations of inflammatory cytokines, alters antigen presentation, and T-cell proliferation

Bowel and Bladder Symptoms refers to Incontinence, urgency, frequency, nocturia and Indicate hyperreflexic bladder. What agents are used for incontinence?

Antimuscarinic agents -Trospium (Sanctura - 40mg/day) -Solifenacin (Vesicare - 5-10mg/day) -Darifenacin (Enablex - 7.5-15mg/day) -Fesoterodine (Toviaz - 4-8mg/day)

NSAIDs treat constitutional symptoms of SLE. Dose should be sufficient to provide anti-inflammatory effects. Consider low-dose aspirin regimen in patients with:

Antiphospholipid syndrome along with anticoagulation Balance risk of renal injury with anti-inflammatory benefit Always discuss risk of GI irritation Caution against OTC duplications of therapy

Atomoxetine (Strattera) is 2nd line after stimulants but considered 1st line in whom?

Anxiety disorder, Severe tics, Risk of misuse or diversion (including by family members) -No DEA Schedule -Initial somnolence and GI upset: Avoid by gradual dose titration

____________________ mostly used pre- and postop for prevention of increased IOP after anterior-segment laser procedures

Apraclonidine (Iopidine)

This type of gout presents as "polyarthritis" and may be confused with osteoarthritis or rheumatoid arthritis?

Atypical gout *polyarthritis = involves 5+ joints*

The following characteristics below describes which medication of Interferon-β1a: -30mcg IM dose once weekly -Prefilled syringes should be refrigerated, but can be at room temp x 30 days

Avonex

Interferon-β1a contains which medications:

Avonex -30mcg IM dose once weekly -Prefilled syringes should be refrigerated, but can be at room temp x 30 days Rebif -Given as either 22 or 44mcg SC three times weekly -Prefilled syringes with autoinjector Plegridy (pegylated formulation of Interferon-β1a) -High-dose, low frequency dosing -Dosed every 14 days

KNOW _______________ requires dose reduction in TPMT deficiency or concomitant allopurinol/febuxostat therapy

Azathioprine (AZA) 2 mg/kg/day PO AE: myelosuppression, HZV infection, cancer, hepatotoxicity, ovarian failure

*QUESTION* EO is a 42 year-old male (5'11", 220 lbs) with chief complaints of a swollen left big toe and extreme pain that began in the middle of the night last night. The area is erythematous and tender. Laboratory analysis reveals a uric acid level of 8.3 mg/dL and SCr 0.9 mg/dL. Review of EO's past medical history reveals dyslipidemia and diabetes. A diagnosis of gout is made. Which is the best choice for the treatment of EO's acute gouty arthritis attack? A. Losartan 100 mg daily x 8 weeks B. Naproxen 750 mg, followed by 250mg q8h until pain lessens C. Allopurinol 100 mg daily, titrating to uric acid <6 mg/dL D. Pegloticase IV every 2 weeks E. Colchicine 1.2 mg stat, followed by 0.6 mg in 1 hour

B

The following characteristics below describes which medication? 10mg po TID and titrated up if needed -Response usually with 40-80mg/day -Continuous intrathecal administration option for pts unresponsive to oral therapy -Do not DC abruptly due to seizure risk -Small doses of diazepam may enhance effect

Baclofen (Lioresal) - preferred agent

What medications are used for the treatment of Gait Difficulties and Spasticity?

Baclofen (Lioresal) - preferred agent Tizanidine (Zanaflex) Diazepam (2-10mg/day) Clonazepam (1-3mg/day) Dantrolene (Dantrium - 100-400mg/day)

Muscle Relaxants are sometimes utilized in addition to pain management therapy. What are the preferred agents (antispasmodics)?

Baclofen 5-20 MG Q6-8h PRN (max 80mg/day) ▪Risk of sedation Tizanidine 2-4 mg Q6-8H PRN (max 36 mg/day) ▪Risk of QT prolongation ▪Central alpha-1 agonist: hypotension, dizziness

The following characteristics below describe which medication? MOA: Cap-dependent endonuclease inhibitor resulting in interference with viral RNA transcription and blocked viral replication Activity against influenza A and B

Baloxavir marboxil (Xofluza) 40 mg: Patients 40 kg -<80 kg 80 mg: Patients ≥ 80 kg Two tabs; one day 12 years or older; same dose as adults -Avoid co-administering with polyvalent cation products (laxatives, antacids, supplements, dairy products) -Will be watching for resistance

Creatinine Clearance Example Calculate CrCl for a 26 month old girl. height = 91 cm SCr = 0.27 mg/dL

Bedside Schwartz equation [0.413 x height (cm)] / SCr (mg/dL) = [0.413 x 91] / 0.27 = 139 ml/min/1.73 m2

-Lower IOP by 20-30% -Decrease production of aqueous humor by the ciliary body -Usually needs twice daily dosing The following characteristics above describe which medications?

Beta Blockers Timolol maleate GFS (Timoptic-XE) -Gel forming solution dosed once daily -Shake once before use and wait 10 minutes before using other eye drops Timolol maleate solution (Istalol, Timoptic) -Preservative free (Timoptic Ocudose)

-Autoantibody form reliant on B-lymphocyte stimulator (BLyS) -Fully human monoclonal antibody that prevents BLyS from binding to receptors on B cells, promoting apoptosis -Dose: 10mg/kg IV over 1 hour every 2 weeks for 3 doses then every 4 weeks OR 200 mg SC once weekly The following characteristics above describe what?

Biologic Agent Belimumab (Benlysta) Adverse Effects: N/V, infections, CNS effects

Vaccines result in acquired or also known as adaptive immunity vs. innate immunity (macrophages involved- destroy foreign invaders vs. self) Vaccines contain antigens that can evoke an immune response. Immune response includes what?

Both T cells and B cells. T cells stimulate B cells. B cells produce Antibodies that help the body eliminate the antigen formed. -takes approximately 2 weeks to establish adequate immunity following exposure to the antigen.

__________________ used for long-term mono- or adjunctive therapy

Brimonidine (Alphagan) Use NLO to prevent hypotension

The following characteristics describe which medication? -Boxed Warning & REMS: suicidal ideation & behavior -CI in Crohn's -Indication: Plaque Psoriasis)

Brodalumab (Siliq)

Zanamavir (Relenza®) inhalation has what side effects?

Bronchospasm; sinus symptoms; dizziness; neuropsychiatric events; serious skin reactions

What are considered 2nd line opioid agents?

Buprenorphine Nalbuphine Pentazocine Butorphanol

What is a major warning of TNF inhibitors?

CHF: increased risk of worsening and new-onset CHF Contraindicated in moderate-to-severe CHF (NYHA class III/IV)

-Average increase in HR of 1-2 beats per minute, BP of 1-4 mmHg -Subset of 5-15% of patients with more significant increase The following characteristics describe what?

CV Concerns with Stimulants Before initiation, collect family history -Sudden death, Wolff-Parkinson-White Syndrome, hypertrophic cardiomyopathy, long QT syndrome If positive family history, conduct CV workup of the patient

Guanfacine ER Safety Drug interactions

CYP3A4 inhibitors (ketoconazole, fluconazole) increase guanfacine levels -- Use 1/2 dose of guanfacine CYP3A4 inducers (rifampin, efavirenz) decrease guanfacine-- Use 2X dose of guanfacine

Sensory symptoms usually do not require treatment but is numbness and paresthesia are frequent complaints. Some may develop acute or chronic pain syndromes (Trigeminal neuralgia, painful dsethesias). What is preferred treatment for trigeminal neuralgia?

Carbamazepine Also agents used for neuropathic pain ¤Amitriptyline ¤Gabapentin ¤Pregabalin ¤Duloxetine

Reduce IOP by 15-26% and decrease ciliary body aqueous humor production by blocking sodium and bicarbonate secretion from the ciliary body to the aqueous humor. This describes:

Carbonic Anhydrase Inhibitors - Topical -Dorzolamide (Trusopt) -Brinzolamide (Azopt) -Brinzolamide/Brimonidine (Simbrinza)

*KNOW* What pregnancy category is tazarotene (tazorac)?

Category X *MUST use contraception*

central neuropathic pain:

Caused by brain or spinal cord injury Ex: stroke, ischemia, MS

Peripheral neuropathic pain:

Caused by nerves outside of brain or spinal cord Ex: diabetic neuropathy, post herpetic neuralgia

Inpatient Empiric Therapy If non-severe penicillin allergy:

Ceftriaxone 50 - 75mg/kg/dose IV divided q12 - 24h (max daily dose: 2000 mg) Severe penicillin allergy Levofloxacin 6 month - < 5 years: 10 mg/kg/dose q12h > 5 years: 10 mg/kg/dose q24h (max daily dose: 750 mg)

Which of the following NSAID is COX-2 selective & is CI in patients allergic to sulfonamides?

Celecoxib GI safety benefit is lost at doses >200 mg/day No difference in efficacy Vs. NSAIDs

_______________ at doses higher than 200 mg per day may be linked with an increased risk of serious side effects related to heart and stroke, especially in patients with pre-existing factors for heart disease or those taking the drug for longer than 18 months

Celecoxib -CYP2C9 poor metabolizer DOSE: 200-400mg x1, then 100-200mg PO BID Max = 400mg/day

Uncomplicated Urinary Tract Infections FIRST LINE TREATMENT OPTION:

Cephalexin (3-5DAYS) Efficacy: E.coli urine isolates have high sensitivity and high drug levels in urine Safety: narrow spectrum (↓ resistance) and very well tolerated

Which of the following TNF inhibitor is a "pegylated humanized antibody fragment"?

Certolizumab (Cimzia): 400mg SQ at Week 0, 2, 4, then 200mg every other week

Polio vaccine routine vaccine for children - protects against polio enterovirus which can cause paralysis. What are dosing administration:

Children (2, 4, 6 to 18 months, 4 to 6 years) - IM, SQ

Who should be vaccinated with PCV13 series?

Children 2mos and older Series: 2, 4, 6 mos; booster at 12-15mos

Match the following description to either acute or chronic gouty nephropathy. -Long term deposition of urate crystals -Microtophi may form -Decrease in kidneys' ability to concentrate urine

Chronic gouty nephropathy

Lupus Nephritis All patients should receive hydroxychloroquine to reduce damage and flares

Class 1: histologically normal on light microscopy but with mesangial deposits on electron microscopy Class 2: responds to treatment w/ corticosteroids Class 3: responds to treatment high dose corticosteroids Class 4: corticosteroids and immunosuppressive agents Class 5: extreme edema and protein loss

The following medications should be cautioned with a reduced dose for hepatic impairment. (Fentanyl, morphine; Hydromorphone; Hydrocodone; Methadone, buprenorphine; Oxycodone) which medications are not recommended?

CoMeT + oxymorphone Codeine; Meperidine; Tramadol-- for renal as well

The following characteristics below describe which opioid morphine like agent? -Only combination products available for pain -Same level of side effects as morphine, but less effective: not preferred -Metabolized to active metabolite morphine via CYP2D6 -C-II alone; C-III combos (ie, tylenol #2, 3, 4); C-V cough syrup

Codeine Ineffective in poor 2D6 metabolizers

Beta Blocker Combinations include:

Combigan -- Brimonidine Cosopt, Cosopt PF (preservative-free) -- Dorzolamide Xalacom -- Latanoprost

Which side effect of opioids do not go away?

Constipation Senna-S (sennosides + docusate) 2 tablets PO qHS Side effects are less prominent in patients with severe pain! & tolerance to side effects develops over time....except for constipation!!!!

Lupus Nephritis Induction Regimens

Continue induction therapy for 6 months unless proteinuria or SCr worsens by 50% or more at 4 months After induction, patients who improved can be maintained on mycophenolate or azathioprine with low doses of corticosteroids if needed

Avoid use for mild symptoms and use them in severe cases such as CNS disease, pneumonitis, vasculitis, thrombocytopenia. This describes what?

Corticosteroids Mild disease: 7.5 mg - 20 mg prednisone daily (max goal is 7.5 mg/day) Mild flare: 0.25-0.5 mg/kg/day Severe disease: 1-2 mg/kg/day In very serious active disease, use IV pulse therapy of IV methylprednisolone 250-1000 mg daily for 1-3 days

Fibromyalgia consists of a widespread musculoskeletal pain, fatigue, insomnia, +/- cognitive symptoms. Treatment options associated with this condition include (Patient education, Exercise, Medications). What are other options?

Cyclobenzaprine ▪Sometimes used as alternative to TCA in mild-moderate symptoms Venlafaxine, SSRIs Gabapentin Tramadol (Opioids not recommended) Non-pharmacological ▪Cognitive behavior therapy (CBT) ▪Referral to specialist ▪Acupuncture, yoga????

Used in severe organ- or life-threatening disease or when patient doesn't respond to other immunosuppressive:

Cyclophosphamide (CYC) monitor for WBC toxicity require premedication for CYC-induced N/V

What are key inflammatory mediators?

Cytokines: TNF, IL-1, IL-6 Soluble Mediators: Prostaglandins, Leukotrienes, Matrix metalloproteinases

Definitions per the Guidelines

DMARD monotherapy -Most often defined as the use of MTX monotherapy, but may also be SSZ, HCQ, or LEF Double DMARD therapy -MTX+SSZ, MTX+HCQ, SSZ+HCQ, or combinations with LEF Triple DMARD therapy -MTX+SSZ+HCQ DMARD combination therapy -Double or triple traditional/conventional DMARD therapy

Systemic Beta Blocker AE:

Dec. HR/BP, negative inotropic effects, conduction defects, bronchospasm, CNS effects, lipid alterations, masked hypoglycemia -IOP reduction may be lower: pt also takes PO beta blocker -1-drop of timolol 0.5% in each eye: similar to 10 mg PO

When to start treatment with DMTs?

Determination of: Severity of illness, Efficacy of medication, Side effects, Cost -Slow the course of illness, but do not suppress completely -Some pts won't have apparent benefit -Evidence that most untreated pts with have progressive disease over time -Very early therapy is effective

This is the active isomer of methylphenidate -Double potency of methylphenidate -If patient is taking methylphenidate, use ½ dose of...

Dexmethylphenidate Focalin IR - BID without regard to meals Focalin XR - once daily °50% IR beads and 50% ER beads °May sprinkle on applesauce

Amphetamines preferred for use in adults, they include:

Dextroamphetamine/ Amphetamine °Adderall IR °Adderall XR - once daily taken whole or sprinkled on applesauce Lisdexamphetamine °Vyvanse: Capsule, chewable tablet °Prodrug: l-lysine bonded to dextroamphetamine hydrolyzed in the blood °Lower abuse potential due to slower onset of effect

Pseudobulbar Palsy caused by progressive degeneration of corticobulbar tract in pts with MS have dysarthria, dysphonia, and dysphagia. What meds. are used for treatment? Sudden, uncontrollable, emotional outbursts such as crying or laughing inappropriately

Dextromethorphan/quinidine 20mg/10mg is used for the treatment of this pseudobulbar affect -One capsule po daily for 1 week, following by one capsule po bid -Mechanism unknown

The following characteristics below describes what? -Synovitis of at least one joint and no other explanation -Positive laboratory tests including RF, ACPA, CRP, and ESR -Duration of symptoms more than or equal to 6 weeks -Criteria: ≥ 6/10

Diagnosis of RA

Peramivir (Rapivab®) IV is infused over 15-30 min. and has dose adj. for renal impairment. What are the side effects?

Diarrhea; neuropsychiatric events; serious skin reactions

Varicella-Zoster Virus is caused by a virus that results in fever, malaise, and pruritic vesicles that crust over. Where does this virus remain dormant?

Dorsal ganglia and can be reactivated as herpes zoster (shingles) -Reasons for reactivation are not certain, but may include age and immunosuppression

KNOW Hydralazine doses over 200mg/day or cumulative dose greater than 100g Results in what?

Drug-Induced Lupus -More common in slow acetylators -Rapid resolution upon discontinuation of offending agent (Some pts require corticosteroids or topical agents) Most common causes of drug-induced lupus include hydralazine(5-8%), procainamide(20%), and quinidine

Patients with ADHD are at increased risk for:

Early death, suicide, increased psychiatric comorbidity, increased substance abuse, lower educational achievement, increased rates of incarceration

Corticosteroids may cause dose & duration dependent AE's. These AE's include:

Elevated BP & BG

What is Pseudo-exacerbation?

Episode with symptoms of exacerbation, but caused by something else (Heat, infections, stress, etc) Rule out before treatment initiated or DMTs altered

This type of psoriasis presents as: -erythema covering nearly entire BSA -serious, life threatening -skin on most of the body looks burnt -hypothermia -fluid loss & dehydration -generally preceded by another type of psoriasis

Erythrodermic psoriasis

*KNOW* Which of the following TNF inhibitors was the first to be studied in pediatrics?

Etanercept (Enbrel): 50 mg SQ twice weekly for 12 weeks, then 50 mg weekly thereafter rebound does NOT occur when discontinued*

*KNOW* How often is SCr monitored in patients taking cyclosporine?

Every 2 weeks x3 months then once monthly

Chondroitin is a mucopolysaccharide found in most mammalian cartilaginous tissues thought to work by providing substrate for joint matrix formation and inhibition of enzymes that break down old cartilage. What is this medication used for?

Evidence is and one study showed benefit for hand OA 800-2000 mg once daily or in divided doses -Slow acting symptom relief - give a trial of 3 months -Very little safety data - mild GI upset -Drug interactions - anticoagulant, antiplatelet

SUBCUTANEOUS (SUBCUT) VACCINE ADMINISTRATION

Example Vaccines: Measles Mumps Rubella (MMR) Varicella (VAR) Inactivated Polio (IPV)* Pneumococcal (PPSV23)* *Can be SUCUT OR IM

INTRAMUSCULAR (IM) VACCINE ADMINISTRATION

Example Vaccines: COVID-19 Influenza IIV Pneumococcal PCV 13 DTaP and Tdap Zoster (RZV) Hep A and B HPV

Reassess the risk/benefit for patients who develop:

Exertional chest pain, Unexplained syncope, Arrhythmias Check regularly: BP, HR

RDS Pharmacologic Management include what?

Exogenous surfactant replacement -Reduces mortality and respiratory morbidity Inhaled nitric oxide -May improve oxygenation by vasodilation of the pulmonary vasculature

Know What are the most common symptoms of Lupus?

Fatigue and Arthritis / Arthralgia -All joints may be affected -Constitutional: Fatigue, Fever, Anorexia, wt loss

The following AE below refer to which DMT medication? Pronounced first dose bradycardia -Hourly pulse and BP monitoring for 6 hours -Must do this for all pts starting treatment -ECG monitoring prior to dosing and at the end of the 6 hrs High risk pt?: extend time period, consider overnight

Fingolimod (Gilenya) Required if pt is restarting therapy or has certain pre-existing conditions -QT interval prolongation -Drug interactions

INFLUENZA VS. COLD VS. COVID-19 SYMPTOMS

Flu comes on really hard and usually no GI issues in adults with no taste or smell Covid usually has lost of taste of smell and GI issues in adults Cold starts with sore throat and goes into nasal issues due to congestion, may get some lost of taste and smell due to congestion

Which of the following medications pertain to the following characteristics below? Dosing (IR): 300mg PO TID: 1,800-3,600 mg/day ▪Widely accepted for all types of neuropathic pain ▪FDA-approved for postherpetic neuralgia (PHN) and seizures Gralise and Horizant - ER formulations ▪No indication for seizures - only neuropathic pain (PHN)

Gabapentin (Neurontin)

Anticonvulsants used for neuropathic pain (frequently off-label) ___________ and __________ have the most evidence. ▪Both require renal adjustments!

Gabapentin; pregabalin Monitor ALL patients being started on anticonvulsants for s/sx of increased suicidality/depression TAPER off - do not abruptly discontinue

Pain Assessment:

General patient history -PMH, HPI, meds, social hx, family hx, allergies Clinical exam -Pain response, physical exam, imaging Etiology -Source of pain (or no physiological cause!) Pain evaluation -Pain attributes interview, tools, and scales -What type of pain are we treating?

_______________ can be given either IV or SubQ.

Golimumab (Simponi): 50 mg SQ once monthly

This type of psoriasis presents as: -"dew-drop like" -small bumps -may clear & never return OR may persist for life -URI often precedes by 2-3wks

Guttate psoriasis

RA lasts 30min-All day and mainly affects the what although it progresses to more joints as inflammation continues?

Hands, feet, Wrists Persistent inflammation and proper exercise is diminished → Joint deformity

Boxed Warning of stimulants include what?

High potential for abuse and dependence -Assess the risk of abuse prior to prescribing and monitor for signs of abuse and dependence

-Dose: 200 mg PO BID or 400 mg PO daily (take w/ food) Max safe dose is 5 mg/kg actual body weight -Therapeutic effects in 2-8 wks, max clinical efficacy: 3-6 months The characteristics above describe which medication?

Hydroxychloroquine (HCQ) Adverse effects typically mild: GI/skin reactions -Retinal toxicity (<1% at 5 years and <2% at 10 years) -Begin annual retinal exams after 5 years of therapy

The following characteristics below describe which medication? -Anti-malarial -MoA unknown: Reduces antigen-antibody response at site of inflammation perhaps by affecting cellular migration and activation -Onset of action up to 6 weeks -Treatment failure only if no response > 6 months -Place in therapy d/t NO myelosuppressive, NO hepatic and renal toxicities

Hydroxychloroquine (Plaquenil)

What meds. should be used cautiously in pts with significant depression?

IFN and natalizumab -Depression Common in MS: Risk of suicide increased -Monitor pts closely -Treat with antidepressants

INFLUENZA VACCINE DURING ANTIVIRAL TREATMENT?

IIV and RIV is okay during treatment and chemoprophylaxis LAIV should not be used until 48 hours after therapy is complete If antiviral therapy is given within 2 weeks after a dose of LAIV, the LAIV should be repeated 48 hours after the antiviral medication is completed

Influenza Vaccine - Indications and Timing

INDICATED FOR...EVERYONE!!! (≥6 MONTHS!) - Ideal administration window is September and October -Goal: vaccinate by the end of October -July and August may be too early -Continue to vaccinate until the vaccine expires

CF-related diabetes mellitus (CFRD) treatment is what?

INSULIN Nutrition also plays a role

Age 4-5: Methylphenidate DOC Rate of metabolism is slower in this age group Start low, go slow. What is the dosing?

IR: 2.5 mg by mouth twice daily Titrate to 7.5 mg 2-3 times daily and up to 10 mg three times daily Methylphenidate: FDA-No; AAP-Yes Dextramphetamine: FDA-Yes; AAP-No

Infliximab (Remicade) is a chimeric monoclonal antibody (mouse and human) indicated for adults with moderate to severe active rheumatoid arthritis with methotrexate. What is the dose of this medication?

IV 3 mg/kg at 0, 2, and 6 weeks, followed by 3 mg/kg every 8 weeks thereafter -Response seen in 4 weeks -Infuse over 2 hours -Begin infusion within 3 hours of reconstitution May lose efficacy due to antibody development (murine component) -- 14-40% of patients

Hospitalized patients who are unable to tolerate oseltamivir or absorb it due to known or suspected gastric stasis, malabsorption, or GI bleeding should consider:

IV peramivir No clinical benefit was demonstrated in a trial looking at patients ≥6 years, but it was well tolerated Monitor for secondary bacterial pneumonia Monitor fluid status

Tocilizumab (Actemra) You must monitor neutrophils / platelets and don't initiate among pts with ANC <2000, PLT <100,000 or LFT >1.5x ULN. How can this medication be initiated?

IV: 4 mg/kg every 4 weeks; may inc. to 8 mg/kg once every 4 weeks based on clinical response (max: 800 mg) SubQ: <100 kg: 162mg once every other wk; inc. to 162 mg once every week based on clinical response ≥100 kg: 162mg once every week

The following characteristics below describe what? Motrin, Advil; IV: Caldolor -OTC: 200 mg -RX: 400mg, 600mg, 800mg Various combination products -Duexis (w/ famotidine) -Vicoprofen (w/ hydrocodone) Commonly used in pediatrics -5-10 mg/kg/dose (antipyretic); max 40 mg/kg/day

Ibuprofen

Recommendations on use a treat-to-target strategy over a non-targeted approach:

In patients who have never taken a DMARD with low, moderate, or high disease activity, use DMARD monotherapy over double and triple therapy -MTX If disease activity remains moderate to high with DMARD monotherapy, add a DMARD (double DMARD), or a TNFi or a non-TNF biologic

Opioids: Starting Doses

In situations with intermittent pain Ineffective pain relief: Increase dose of short-acting opioid PRN Dose "runs-out" towards the end: Keep dose the same, reduce dosing interval if possible (ie, Q6 -> Q4) In situations with persistent, around-the-clock pain Add long-acting scheduled opioid ▪Continue short-acting PRN opioid for breakthrough pain Titrate long-acting opioid based on amount of breakthrough doses used ▪Increase long-acting agent by 25-100% of the total daily breakthrough dosing used based on severity of pain

Atomoxetine (Strattera) boxed warning is what?

Increased risk of suicidal ideation in children or adolescents Monitor: Suicidal thinking or behavior, Clinical worsening, Unusual behavior changes Study duration 6-18 wks (pooled analysis of 2200 patients) -Early in treatment -Suicidal ideation 5/1357 (0.4%) v. 0/851 in placebo -No Suicides occurred

The following characteristics below refer to which influenza stage? -Typical incubation period is 1-7 days; 2 days = average -Adults can be contagious before symptoms

Incubation

Which medications must be given with methotrexate?

Infliximab (Remicade) Golimumab (Simponi)

*KNOW* Which of the following TNF inhibitors is the ONLY one administered via IV infusion (infused over 2hrs)?

Infliximab (Remicade): 5 mg/kg over 2-3 hours IV at week 0, 2, and 6; then every 8 weeks *all others are SubQ*

Dimethyl fumarate (Tecfidera) has a dosage of what?

Initial: 120mg (delayed release) po bid After 7 days: 240mg (delayed release) po bid

This type of psoriasis usually develops as lesions in skin folds (area where skin touches skin - armpits, genitals, crease of buttocks)?

Inverse psoriasis *aka intertriginous psoriasis or flexural psoriasis*

The following characteristics describe which medication? -Humanized IgG4 monoclonal antibody -Indication: plaque psoriasis & psoriatic arthritis

Ixekizumab (Taltz)

This medication is indicated for the management of moderately severe acute pain that requires analgesia at the opioid level should be used for a max of 5 days to reduce GI bleeding risk and includes both IV/oral combined.

Ketorolac Oral - only approved for continuation of IV/IM therapy

*KNOW* Corticosteroids have __________ upset than NSAIDs.

LESS GI

Special Populations

LOW BACK PAIN Acute/sub-acute pain Non-pharmacological treatment (heat, massage, etc.) Rx: NSAIDs or skeletal muscle relaxants ▪Opioids > placebo (pain), but = to celecoxib or antidepressants CHRONIC Strong recommendation: non-pharmacological treatment ▪Best evidence (moderate-quality) for exercise, rehab, acupuncture, mindfullness-based stress reduction If failing non-pharmacological:Rx ▪1st line: NSAIDs ▪2nd line: Tramadol or duloxetine ▪Last line: opioids

For GI risks associated with ulceration, treatment/prophylaxis of PPIs such as what may be of use?

Lansoprazole 15 or 30 mg daily Esomeprazole 20 or 40 mg daily ◦Prostaglandin analogs -misoprostol/diclofenac (Arthrotec)

_____________________________ has the following characteristics below: -Once daily dosing -IOP lowering greater than latanoprost -Increased incidence of ocular AE: Ocular pain and hyperemia

Latanoprostene Bunod (Vyzulta) Nitric oxide-donating prostaglandin analog -Nitric oxide increases trabecular meshwork outflow -PG analog effect increases uveoscleral outflow

___________________ is a ProDrug, inhibits pyrimidine, has a long t1/2, and has a loading dose of 100mg QD x3 then maintenance dose of 20mg QD.

Leflunomide (Arava) -Any disease duration / severity -Can decrease if SE -w/o loading dose, steady state takes MONTHS to achieve

Pancreatic enzyme dosing is based on what?

Lipase

ADVERSE REACTIONS of Influenza vaccine

Local - Redness, muscle soreness Mild - Fever, malaise Severe - Allergic reaction, Guillain-Barré Syndrome (GBS) (may appear up to 6 weeks after) More local reactions with high-dose and adjuvanted (Fluad) Flu-like symptoms more likely with LAIV Congestion, sore throat, headache, fever

-Useful as second-line monotherapy or adjunctive therapy -Dosed Q8 or Q12 hours What are the AE of Carbonic Anhydrase Inhibitors - Topical?

Local effects - burning, stinging, transient blurred vision, tearing, photophobia, rare conjunctivitis, bitter taste Few systemic effects - sulfa allergy

Opioids have no true max dose and varies per patient. What should be used for persistent pain?

Long-acting (LA) opioid + IR opioid PRN breakthrough pain ▪IR opioid should be dosed at ~10-15% of total 24 hour dose of LA opioid

A disease of autoantibody formation, first described in 13th century by erosive lesions. This disease is recognized as a systemic inflammatory syndrome since 1800s:

Lupus

-An ACE inhibitor or ARB should be given to reduce proteinuria by about 30% or 0.5 g/day and delay progression of renal disease -Pts w/ LDL >100 mg/dL should receive a statin to prevent accelerated atherosclerosis The following characteristics above serve for what?

Lupus Nephritis Maintain BP < 130/80

Presumed Atypical Bacterial Pathogens:

Macrolide, with azithromycin being preferred

Angle Closure Glaucoma is often a Medical Emergency that one should Control symptoms with?

Medications or Iridotomy (surgical or laser hole in the iris)

-Inhibitor of dihydrofolate reductase (DNA synthesis & cell proliferation) -Use primarily pts w/ arthritis & skin disease as a steroid-sparing therapy -Toxicities reduced by folic acid administration The following characteristics describe which medication?

Methotrexate (MTX) Dosed: once weekly at range of 15-25 mg/week PO or SC

The following dosing characteristics below describes which medication? -Start 7.5 mg once weekly -Typical 10 to 15 mg once weekly -Increase by 5 mg every 2 to 4 weeks to a maximum of 20mg (30 mg) once weekly -PO doses >15 mg, may want to switch routes d/t BA -Alternate dosing: 2.5 mg PO q 12 hours for 3 doses every week

Methotrexate (Rheumatrex, MTX) -- PO, SQ, IM -Add folic acid 1mg PO daily -Consider IM/SQ if GI side effects

Non-biologic DMARDs for RA include:

Methotrexate (Rheumatrex, MTX) -- PO, SQ, IM Leflunomide (Arava) - ProDrug Hydroxychloroquine (Plaquenil) Sulfasalazine -- ProDrug

This DMARD is known as the initial of choice with anti-inflammatory action, fast onset, with a pregnancy cat. X and no for lactation.

Methotrexate (Rheumatrex, MTX) -- least expensive 2-3 wks for initial effect, improv. up to 12 CI: CrCl < 40ml/min, liver dx, immunodef., pleural or peritoneal effusions, leukopenia, thrombocytopenia, preexisting blood disorders

Apnea of prematurity is treated with _______________ which include Theophylline/Aminophylline & Caffeine. What is their Mechanism of action?

Methylxanthines •CNS stimulant •Increase respiratory drive •Increase sensitivity to hypercapnia •Increase diaphragmatic contractility

AE of Glatiramer acetate (Copaxone, Glatopa):

Mild side effects -Pain and pruritus at injection site -One-time transient reaction: chest tightness, flushing, and dyspnea -Significant reduction in annual relapse rate (~29%) -May slow progression of disability in pts with RRMS -Delays development of black holes on brain MRIs -Remains safe and effective beyond 10 yrs

This medication is Approved for reducing neurologic disability and frequency of clinical relapses in pts with SPMS, PRMS, or worsening RRMS:

Mitoxantrone (Novantrone) Administration -5 to 15 minute IV infusion dosed at 12mg/m2 q 3 months -Evaluation of left ventricular EF and ECG are required prior to each dose or if signs of CHF develop -Max allowable lifetime cumulative dose = 140mg/m2

Glucosamine is an endogenous substance made from glucose and used in biosynthesis of proteoglycans and glycosaminoglycans, the building blocks of cartilage. Recent guidelines recommend against use, what is this medication used for?

More evidence for sulfate salt dosed 1500 mg daily Full effect seen after 2 months of use SEs - abdominal pain, heartburn, diarrhea Caution in DM for glucose elevation

The following characteristics below describe which opioid morphine like agent? -Opioid prototype - often used for severe pain -Venous and arteriolar vasodilation ▪Commonly used in MI patients -Renally-cleared (not recommended): watch for accumulation of metabolites (side effects)

Morphine

Drug Causes: Anticholinergic Eye Drops Mydriatic/cycloplegic agents used to dilate the eye for routine eye exams:

Most anticholinergic: Atropine, Homatropine Mydriatic w/ less anticholinergic & less inc. IOP -Tropicamide -Phenylephrine -Inc. IOP and can precipitate angle-closure glaucoma -IOP is measured before use of the agents

Abrupt ADHD medication discontinuation increases risk for:

Motor vehicle crashes, criminality, drug-related crimes, violent reoffending, depression, interpersonal issues, injuries

CI in pregnancy and Most studied in treatment of lupus nephritis describes which medication?

Mycophenolate Mofetil 2-3 g/day PO Lower doses needed in Asians; may be more effective than cyclophosphamide in African Americans and Hispanics

A single dose ≥7.5 g is likely toxic for acetaminophen. Overdose is assessed with Rumack-Matthew nomogram although you will be asymptomatic within 24hrs of ingestion. How do you treat in case of overdose?

N-acetylcysteine (NAC) immediately (most effective within 8hrs.) Can consider activated charcoal within 4 hours of ingestion: 1g/kg (max 50g) PO x1

Oseltamivir (Tamiflu®) capsule; susp. has what side effects?

N/V, headache; neuropsychiatric events; serious skin reactions

Understand what NAS (Neonatal abstinence syndrome) is and approach to treatment:

NAS is a screening for Intrauterine Drug Exposure •Maternal characteristics that prompt screening of the neonate include: -History of drug abuse -Late or no prenatal care -Mood swings -Many spontaneous abortions Methods for screening -Urine -Meconium

UVA or UVB is used as light therapy to treat psoriatic lesions. UVB given alone as either broadband (BB-UVB) or narrowband (NB-UVB). Which of the 2 is the preferred method?

NB-UVB

What are the supportive therapies used for RA?

NSAIDs Corticosteroids

Any type of psoriasis that may involve the nails ("nail pits" - tiny dents in the nails)?

Nail psoriasis

The following characteristics below describe what? Aleve, Naprosyn, Anaprox, Naprelan -OTC: 220mg naproxen sodium -RX: 375mg-550 mg (IR, ER, DR) Various combination products -Treximet (w/ sumatriptan) -Vimovo (w/ esomeprazole)

Naproxen 200 mg naproxen base = 220 mg naproxen sodium

NASAL VACCINE ADMINISTRATION

Nasal syringe, no needle, take off rubber protector, spray in one nostril, clip will stop you, take off clip and repeat in other nostril rapidly as possible

Theophylline & Aminophylline: Adverse Reactions

Nausea, vomiting, tachycardia, agitation Severe: arrhythmias, hypotension, seizures, death

The following characteristics below refers to whom: ►Live vaccines are not administered until maternal antibodies have waned ►Generally by 12 months of age ►Premature & breastfed infants use standard schedule & doses

Neonates and Infants

*KNOW* The MOST common AE of cyclosporine is?

Nephrotoxicity

NSAIDs are CI in pregnancy, used for mild-mod. pain/inflammation (HA, Menstrual, Dental, Perioperative) and is not effective for what?

Neuropathic pain Switch to NSAID of another "class" if ineffective after ~1 month trial

BB agents include:

Nonselective Timolol -- Betimol, Timoptic-XE, Istalol Levobunolol -- Betagan Metipranolol -- Optipranolol B1-selective Betaxolol -- Betoptic S Nonselective with ISA Carteolol -- Ocupress

Pain Assessment: Pain Scales

Numeric rating scale (NRS) Standard scale 0-10 from no pain ==> severe Behavioral pain scale (BPS) -Non-verbal! Used with patients who cannot respond (ie, mechanically ventilated) -Looks for facial expression, limb movements, other physical characteristics Visual analog scale (VAS)

This medication is first & ONLY approved for BOTH relapsing and primary progressive MS and CI in pts w/ active Hep B infection:

Ocrelizumab (Ocrevus) Infusion reactions - can be severe AE: U/LRTI, skin infections, infusion reactions

Netarsudil (Rhopressa) Rho kinase inhibitor increases outflow of aqueous humor through the trabecular meshwork has IOP characteristics as such: -IOP <25mmHg: similar lowering to timolol -IOP >25mmHg: better than timolol What is the dosing?

One drop once daily in the evening -Conjunctival hyperemia in 50% -Prescription drug label -Combination netarsudil and latanoprost (Rocklatan)

The following counseling points below describes which medication? -Take on an empty stomach -Avoid use with alcohol due to risk of rapid release of the drug

Opana ER

Chronic airway inflammation Treatment:

Oral Azithromycin

ORAL VACCINE ADMINISTRATION include which of the following:

Oral liquid - example Rotavirus •Prepare the tube of vaccine per package insert •Squeeze tube of vaccine toward inner cheek of child •Re-administration is not recommended if child spits out vaccine Oral capsules- example Typhoid (live) •One capsule is taken every other day, for a total of 4 capsules. The last dose should be taken at least 1 week before travel. •Each capsule should be swallowed whole (not chewed) about one hour before meals with cold or lukewarm water. •Capsules must be stored in the fridge

Treatment of Pulmonary Exacerbations in CF:

Oral, inhaled or IV antibiotics

Product substitutions cannot be made directly based on strength, check the FDA _________ for AB rated products

Orange Book

Drug of choice treatment for specialty populations

Oseltamivir Pregnancy: Any neuraminidase inhibitor -Avoid baloxavir Immunocompromised -Avoid baloxavir Hospitalized patients with severe or complicated illness: may want to double dose and may treat longer than 5 days

The following characteristics below describe which medication? -MOA: Neuraminidase inhibitors resulting in impaired release of virus from infected cells and decreased viral replication -Activity against influenza A and B

Oseltamivir (Tamiflu®) -- wt based: birth-12; >/=2wks 75mg BID within 48 hrs of symptoms; treat 5 days zanamavir (Relenza®) -- >/=7 same as adult Two inhalations BID within 48 hrs of symptoms; treat 5 days; first 2 doses separated by at least 2 hrs, then Q12H peramivir (Rapivab®) -- 600mg once [>/=2]

RECOMMENDATIONS FOR 2-64 HIGH-RISK with the associated problems below: Immunocompromised: HIV, chronic renal failure, nephrotic syndrome, asplenia

PCV13: 8 wks PPSV23: 5 years, then final dose at age 65

RECOMMENDATIONS FOR 2-64 HIGH-RISK with the associated problems below: CSF Leaks of Cochlear Implants

PCV13: 8 wks PPSV23: final dose at age 65

Gastrointestinal treatment in CF:

Pancreatic enzyme replacement therapy (Creon, Pancreaze, Pertzye, Viokace, Zenpep, Ultresa, Relizorb) Fat-soluble vitamin replacement therapy: KADE

Reversible with dose reduction or D/C, This is known to cause Circulation problems in fingers and toes, Raynaud's Phenomenon:

Peripheral Vasculopathy (intermittent / mild) very rare: digital ulceration, soft tissue breakdown Monitor: -Feeling numb, cool, painful burning -Color change pale to blue to red -Unexplained wounds on fingers or toes

Diagnosis of psoriasis is based on?

Physical examination findings of characteristic lesions No skin biopsies of lab tests

CNCP Non-Pharmacological Tx

Physical rehabilitative therapies -Decreased all-cause mortality (HR:0.81, p = 0.03) Psychotherapeutic interventions -Decreased all-cause mortality (HR: 0.62, p <0.001) -Cognitive behavioral therapy (CBT) and behavioral therapy decreased disability associated with chronic pain ▪Up to 6 months ▪CBT > behavioral therapy

What is the MOST common type of psoriasis?

Plaque psoriasis

The following characteristics below describes which medication of Interferon-β1a: -High-dose, low frequency dosing -Dosed every 14 days

Plegridy (pegylated formulation of Interferon-β1a)

Infliximab (Remicade) ADRs related to infusion involve Pruritus / urticaria, Chest pain, hypotension, SOB. How would you minimize infusion reactions?

Pre-medicate -APAP and diphenhydramine 90 minutes prior to infusion -Corticosteroids Slow rate of infusion OR Give with MTX or other immunosuppressants Cases of hepatotoxicity and flu-like symptoms occur: fever, chills, fatigue, diarrhea, pharyngitis, HA, CNS complications

For Acute Pain you want to Utilize as-needed (PRN) regimens for intermittent/ breakthrough/variable pain, or as pain improves. Oral is __________ over IV when possible.

Preferred Consider around-the-clock dosing where appropriate (ie, persistent pain)

Which of the following medications pertain to the following characteristics below? Dosing: 75mg PO BID; max daily dose 600mg/day -Used for fibromyalgia, diabetic peripheral neuropathy (DPN), PHN, and other types of neuropathic pain -Preferred in patients with spinal cord damage

Pregabalin (Lyrica)

Early signs include abnormally sustained or frequent and painful erections. This can happen in pediatric and adult patients where they have prolonged and painful erections that may require surgical intervention:

Priapism caused by Methylphenidate Occur after established on the medication and after a dose increase °Also during drug withdrawal °Not at the initiation of use of medication

Primary cause of uric acid UNDEREXCRETION is?

Primary Idiopathic hyperuricemia ~90% of gout patients have ↓ in renal excretion*

Diagnosis of Open-Angle Glaucoma:

Progressive optic neuropathy with structural changes to the optic nerve and open anterior angle chamber Suspect -Consistently elevated intraocular pressure (IOP) -Suspicious-appearing optic nerve -Abnormal visual field

RDS Surfactant Place in Therapy includes what?

Prophylaxis -Administered to neonates at high risk of RDS -Occurs in delivery room before onset of RDS Rescue -Administered to neonates with established RDS -"Early" rescue therapy administered within 1-2 hours after birth -"Delayed" or "late" therapy is administered 2 or more hours after birth

Tremor is a cerebellar symptom that is troubling and difficult to control. What medications are used for this symptom?

Propranolol, Primidone, Isoniazid

Adverse effects of Etanercept (Enbrel):

Pruritic injection site reactions (up to 43%) -lasts 3-5 days x 1st month, then better Other ADRs are similar to the TNF profile -Headache (19%) -Infection (up to 81%) -URTI & LRTI Common: 65% and 54%, respectively Monitor baseline & yearly: PPD, CBC, LFTs

The following characteristics below describe this chronic systemic inflammatory disease: •Skin disorder •T-lymphocyte-mediated •Recurrent exacerbations and remissions •Thickened, erythematous, and scaling plaques •Many comorbidities

Psoriasis

What is the MOST common extracutaneous manifestation of psoriasis?

Psoriatic arthritis Perhaps, systemic inflammation enhances insulin resistance, causing endothelial dysfunction, leading to atherosclerosis and coronary events

Exacerbation of preexisting behavior disturbance and thought disorders known to cause mood swings and dysphoria in preschoolers. This is known as what?

Psychiatric Adverse Reactions - Rare Induction of manic episode in patients with bipolar disorder -Assess for risk (history of depressive symptoms; family history of suicide, bipolar, or depression) New psychotic or manic symptoms (hallucinations, delusional thinking) - 0.1%

Parasympathetic innervation from the oculomotor nerve in the brain to the ciliary muscle and sphincter pupillae muscle causes what?

Pupil constriction Parasympathomimetic (cholinergic): miosis Parasympatholytic (anticholinergic) mydriasis & cycloplegia Lacrimal glands are under parasympathetic control for tear secretion

This type of psoriasis presents as: -pus-filled bumps (collection of neutrophils) -usually on hands or feet -painful -can be acute, localized or generalized -if widespread, severe with fever & toxicity

Pustular psoriasis

This topical capsaicin is FDA approved for PHN and administered in MD office as a single 60 min. application to most painful area of body.

Qutenza (capsaicin 8%) -- may apply 4 patches at once -May repeat after 3 months -Pre-treat with topical anesthetic prior to application

Non-Pharmacologic Treatment Options PAIN

RICE Therapy: Best for acute injuries - strains, sprains and contusions •Rest •Ice -- 15-20min. intervals at least 3-4 times a day •Compression •Elevation -- at or above level of heart for 2-3 hrs/day Heat can be used in non-inflammatory injuries

Non-Routine Viral Vaccines include what?

Rabies - RavAvert: 3 doses for prevention or 1 dose + IG post exposure Japanese Ecephalitis (Ixiaro): 2 doses, 28 days apart; complete 1wk prior to travel Yellow Fever - YF VAX; CI w/ sever allergy to eggs/gelatin; SQ, LIVE

The following characteristics below describes which medication of Interferon-β1a: -Given as either 22 or 44mcg SC three times weekly -Prefilled syringes with autoinjector

Rebif

HepB dosing concerns:

Recombivax HB and Engerix B: 3dose: 0, 1, 6mos HEPLISAV-B (Dynavax): 2dose 1mos apart >18y/o Virus — Hepadnaviridae Humans only known host Blood borne pathogen Incubation 45-160 days Frequently asymptomatic until too late

If switching to same AGENT but different formulation, generally ok to do 1:1 MME conversion. otherwise what?

Reduce dose of new opioid by 25-50% of total daily MME

**Must be careful when using concentrated infant drops!!** What's the compared dosing?

Regular - 100 mg/5 mL Conc drops - 50 mg/1.25 mL

NAS Pharmacologic Treatment: Buprenorphine Partial mu-receptor agonist with Long and predictable half-life. what is the neonatal administration?

Requires extemporaneous preparation and administered sublingually Limited evidence suggests decreased length of stay compared to morphine Safety and efficacy need further studies

The following characteristics below describe which medication? -Monoclonal chimeric antibody to CD20 protein found on the cell surface of B lymphocytes -Results in depletion peripheral B cells: B cells takes months to recover -Intermittent therapy possible -Useful in patients who have failed MTX or TNFi's -Combine with MTX for best results -NO LIVE VACCINES

Rituximab (Rituxan)

Chimeric monoclonal antibody directed at the CD20 antigen on B cells. Demonstrated beneficial effects in case reports and open-label trials but not supported in RCTs. This drug is Indicated in organ-threatening disease or in those intolerant or with contraindications to standard immunosuppressants:

Rituximab (Rituxan) Studies support use in varying degrees of disease manifestations and age groups Often combined with steroids

TETANUS IM (The T in DT, Td, Dtap and Tdap) an illness caused by the exotoxin of Clostridium tetani. When should you receive this vaccine?

Same as diphtheria, or after a wound ►Generally given at 2, 4, and 6 months of age - IM -4th shot at age 15-18 months -5th shot at age 4-6 years ►Booster doses given once every 10 years -Given as Td OR Tdap

The following characteristics describe which medication? -Human IgG1 monoclonal antibody -Can do a loading dose -Higher dosing with plaque and psoriatic arthritis -Indication: Ankylosing spondylitis, Plaque psoriasis (moderate to severe), & Psoriatic arthritis

Secukinumab (Cosentyx)

Non-Stimulants include the following medications:

Selective norepinephrine reuptake inhibitor °Atomoxetine (Strattera) Selective alpha-adrenergic agonists °Guanfacine ER (Intuniv) °Clonidine ER (Kapvay)

Clonidine ER Most common adverse reactions as monotherapy:

Somnolence, Fatigue, Irritability, Nightmare, Insomnia, Constipation, Dry mouth Most common AE combination therapy with stimulants: Somnolence, Fatigue, Decreased appetite, Dizziness

Pilocarpine (Isopto Carpine, Pilocar, Pilopine HS) has 20-30% reduction in IOP. What is the dosing?

Started at 0.5-1% 3-4X daily -2% soln Q 6-12 h -4% gel QD

Treatment Overview for Ages 6-11: Prescribe FDA approved meds Best evidence is for _______________.

Stimulants Sufficient/less strong: Atomoxetine, ER guanfacine & clonidine Evidence-based parent and teacher-administered behavior therapy

The following characteristics below describe which medication? -Prodrug -Cleaved by gut bacteria in the colon to sulfapyradine and 5-ASA (IBD)--What happens in SIBO or gut dysbiosis?? -In this case the sulfapyradine is responsible for effect -Response seen in < 2 months -Binds Fe-limits absorption

Sulfasalazine -AE: yellow-orange skin and urine; CI in hypersensitivity

_____________________ innervate dilator pupillae muscle, blood vessels of ciliary body, episclera, extraocular muscles

Sympathetic fibers Sympathomimetics cause mydriasis without affecting accommodation

For Neuropsychiatric Lupus determine if inflammatory, thrombotic, or combined cause for CNS involvement:

Symptomatic therapy with anticonvulsants or antidepressants For inflammatory: use glucocorticoids alone or in combination with immunosuppressives If no improvement, consider salvage treatment: -Plasma exchange -IV immunoglobulin -Rituximab

_____________________ leads to malaise, fatigue, anorexia, nausea, altered taste, depression, decreased libido

Systemic acidosis

Standard analgesics (APAP, NSAIDs, opioids) are generally not effective for Neuropathic Pain. What are 1st line agents?

TCAs (Amitriptyline, nortriptyline) SNRIs (Duloxetine, venlafaxine) Anticonvulsants (Gabapentin, pregabalin) Alternative agents: - Certain opioids (ie, tramadol, tapentadol) - Antispasticity agents (tizanidine, baclofen)

____________ may induce lupus, however, short-term induction therapy with infliximab (Remicade) may confer long-lasting benefits in lupus nephritis

TNF-alpha inhibitors

__________ induces inflammatory cytokines (interleukins), enhances leukocyte migration, activates neutrophils and eosinophils, and induces acute phase reactants and tissue degrading enzymes

TNFα

AE of Short acting beta agonist (SABA) [Albuterol, levalbuterol] in the treatment of asthma exacerbations:

Tachycardia, Diastolic hypotension, Mild hypokalemia Standard of care, first line

Terbutaline: Adverse Reactions

Tachycardia, hyperglycemia, hypokalemia, hypoxemia, rhabdomyolysis Recommended to use: continuous cardiac monitoring due to arrhythmia risk. May see elevation in troponins Mention that terbutaline and albuterol often still given together. Also discuss that it comes in 1 mL vials so very tedious to draw up

Methylphenidate Immediate Release (Ritalin) dose:

Take BID-TID, 30-45 minutes before a meal °Tablet, chewable tablet, oral solution (Methylin) Ritalin LA and Metadate CD: bead-filled capsules QD (MAY BE SPRINKLED ON APPLESAUCE) Concerta: Once daily without regard to meals Tablet case may be seen in stool

Hydroxychloroquine (Plaquenil) is indicated for all disease durations and severities. How should this medication be taken?

Take with food Start: 200-300 PO mg BID After clinical response or 1-2 months, decrease to 200 mg daily or 200 mg BID

Central analgesics (opioid-like) contain tramadol and tapentadol. The following characteristics below describe which medication? Binds to μ-opiate receptor and also inhibits NE uptake Option for neuropathic pain (ER has approval for DPN; Not 1st line!) Dosing -1st day - may administer 2nd IR dose after the first as soon as 1 hour later! -"Claim to fame": lower GI side effects than other opioids -No alcohol with Nucynta ER -Caution in seizure patients

Tapentadol (nucynta, nucynta ER)

What is the only vaccine-preventable disease that is not contagious?

Tetanus If 3 dose series never received, can give tetanus immunoglobulin if a serious wound + tetanus toxoid if different syringes and injection sites are used

During an exacerbation, albuterol is typically given intermittently initiatlliy. If pt not doing well then what?

They may be transitioned to continuous albuterol High doses of 20-30 mg/hr are well tolerated in; even higher doses of 75-100 mg/hr are well tolerated in this patient population Albuterol may be titrated to clinical response which is done via repeated assessment of severity scores

CHEMOPROPHYLAXIS considered 70-90% effective, useful adjunct to vaccination, limit use to prevent resistance. Who should receive it?

Those at high risk for poor outcomes What about everyone else? -Watch closely and initiate treatment early if fever and/or respiratory symptoms start

Viral vector - genetically engineered vaccines. Viruses are weakened so they cannot cause disease. There are two types:

Those that can still replicate within human cells and those that cannot because key genes have been disabled. Ex: of replicating- Ebola Nonreplicating: Johnson&Johnson COVID-19 vaccine

Clonidine extended release (Kapvay) -- Hypotension, Bradycardia, Syncope:

Titrate slowly (0.1 mg QHS, 0.1 mg QAM and 0.1 mg QHS, 0.1 mg QAM and 0.2 mg QHS) °Minimum of one week at each dose °Taper in increments of 0.1 mg every 3-7 days to avoid rebound BP increase

The following characteristics below describes which medication? titrated over 2 to 4 weeks, starting at 4mg po qhs; 2-36mg/day Short-acting alpha-adrenergic agonist -Increases presynaptic inhibition of motor neurons -Comparable efficacy to baclofen

Tizanidine (Zanaflex)

Schedule II opioid - limited to no more than 3 days

To extend to 7 days: MD to write "Acute Pain Exception" To extend beyond >7 days: MD write "Non-Acute Pain" ▪Chronic nonmalignant pain Acute: Better for pain than for functional improvement

The following characteristics below describe which medications? -IL-6 blocker -Moderate to severe active RA -Failed 1 or more TNF inhibitor (DMARDs) -Can increase LFTs, LDL, HTN, GI perforations

Tocilizumab (Actemra) Sarilumab (Kevzara)

Janus Kinase Inhibitors

Tofacitinib (Xeljanz) Baricitinib (Olumiant) Upadacitinib (Rinvoq)

This type of gout is as a result of late complications due to hyperuricemia and can proceed to deformity & joint destruction?

Tophaceous gout *tophi = crystal deposits in soft tissues*

Drug Causes: Corticosteroid -Inc. IOP in 18-36% of the general population -Inc. IOP in 46-92% of those with POAG -Usually after 2-4 weeks of use The Mechanism includes what?

Topical - decrease aqueous humor outflow Systemic - increase aqueous humor production Topical uses -Peri- and post-op to minimize inflammation -Allergic conjunctivitis and other inflammatory conditions

What is our mainstay of therapy for mild-moderate psoriasis?

Topical corticosteroids decreases redness, scaling & itching*

Drug Causes: Sulfa Drugs -Acute angle closure not related to pupillary block -Ciliary body edema and lens thickening -Lens and iris are pushed forward -Anterior chamber is narrowed Medication involved is what?

Topiramate Also trimethoprim-sulfamethoxazole, acetazolamide, HCTZ Presents as blurred vision and reversed by drug discontinuation

Summary of Drug Distribution:

Total body water: Inc. in neonates & infants ↑ Vd of hydrophilic drugs Total body fat: Reduced in neonates & infants ↓ Vd of hydrophobic drugs Protein binding: Lower amounts & binding affinity in neonates ↑ free fraction of protein bound drugs; displacement of bilirubin (BBB) permeability: Higher in neonates & infants ↑ drug distribution into BBB

Central analgesics (opioid-like) contain tramadol and tapentadol. The following characteristics below describe which medication? -Binds to μ-opiate receptor but also inhibits NE and 5-HT uptake -Pharmacologically active metabolite requires conversion via CYP2D6 -Decrease dose in renal/elderly -Elevated seizure risk -May increase INR -Schedule IV federally as of 2014

Tramadol (Ultram, ultram er, conzip)

The following characteristics below describes what medication? Extended release 100 mg once daily increase after 5 days by 100 if needed -Start with IR to determine the need for ER -Max of 300 mg/day -Use extreme caution in elderly

Tramadol (Ultram®)

The following characteristics below refer to which influenza stage? -Large-particle droplets -Airborne transmission

Transmission

Comorbid Conditions With other neuropsychiatric disorders (anxiety, depression, autism spectrum, Tourette's)

Treat the most impairing disorder first With Bipolar disorder: Stabilize mood with lithium, anticonvulsant, or atypical psychotic first

What is the Hallmark treatment of early RRMS?

Treatment of Exacerbations Treatment indicated -Multiple symptoms present -Relapses localized to optic nerve, spinal cord, or brainstem -Functional limitations affecting ADLs -Symptoms that worsen over period of 2 weeks

A "biosimilar" is a biologic product that is extremely similar to an approved reference biologic and doesn't not have clinically significant differences in efficacy, adverse effects, or immunogenicity compared to the reference product. A. True B. False

True

NSAIDs are contraindicated for the treatment of peri-operative pain in the setting of coronary artery bypass graft (CABG) surgery. A. True B. False

True

Patients with a history of peptic ulcer disease should consider the use of a PPI when using NSAIDs. A. True B. False

True

Pegloticase (Krystexxa) is a PEGylated recombinant uricase that is considered last resort for refractory gout. A. True B. False

True

Providers are encouraged to integrate biosimilars into their practices with the goal of quality health care at lower cost to patients A. True B. False

True

T/F: D5W means 5 g of dextrose in 100mL of water D10W means 10 g of dextrose in 100mL of water NS (0.9 %), ½ NS (0.45 %), ¼ NS (0.2 %)

True

T/F: "Asthma is a heterogeneous disease characterized by chronic airway inflammation. It is defined by the history of respiratory symptoms such as wheeze, shortness of breath, chest tightness and cough that vary over time and in intensity, together with variable expiratory airflow limitation."

True

T/F: 0.12% saw pain specialist on the survey of chronic pain ft. 690 million people

True

T/F: All vaccines aim to expose the body to an antigen that won't cause disease, but will provoke an immune response that can block or kill the virus or bacteria if a person becomes infected.

True

T/F: Bulb syringes: squeeze the ball BEFORE inserting it. Slowly release the pressure to draw out fluid.

True

T/F: Diclofenac may have higher CV risk, If GI risk is low but CV risk is high, choose naproxen.

True

T/F: Fentanyl and methadone appears safest with renal impairment.

True

T/F: Local anesthetics including cocaine usually have rash (within 72 hrs) and anaphylaxis (within 1hr)

True

T/F: Long-Term Suppression of Growth have been inconsistent

True

T/F: Meperidine is not recommended and known to cause seizures?

True

T/F: Mycophenolate Mofetil may be more effective than cyclophosphamide in African Americans and Hispanics

True

T/F: Patient-controlled analgesia (PCA) pump ▪Better pain control ▪Improved patient satisfaction ▪Similar side effects

True

T/F: Risk of serotonin syndrome when combined with other serotonin meds including duloxetine

True

T/F: Rituximab (Rituxan) is Indicated in organ-threatening disease or in those intolerant or with contraindications to standard immunosuppressants

True

T/F: The peak influenza season is in February

True

T/F: memory B and T cells help provide long-term immunity because they continue to circulate in the body, monitoring for subsequent exposures to the same antigen.

True

TNF inhibitors are contraindicated in moderate to severe CHF (NYHA class III/IV) A. True B. False

True

T/F: Hydroxychloroquine (HCQ) is Primarily renal elimination, tissue half-life of 40-50 days and blood levels can be monitored

True -Begin annual retinal exams after 5 years of therapy

T/F: Lupus present different in men / women and can involve almost any organ and can present in many ways.

True -Difficult diagnosis due to many possible etiologies for symptomology -Extensive workup to determine extent of involvement -Average time from patient seeking treatment to diagnosis 9-12 months -Disease manifestations fluctuate with periods of remission, flares, and progression

T/F: Of those with the diagnosis, it will continue into adolescence for 75% and into adulthood for 50%. In kids the ratio of male:female is 2:1, in adults is 1.6:1

True -Prevalence: 7-8% of children and youth -35.5% inc. in medication use in 2012 compared to 2008 -The number of adults taking medications inc. by 53.4%

T/F: Belimumab (Benlysta) is Indicated in patients uncontrolled on standard therapy, 5 years of age and older

True Approval trials found no benefit in patients of African heritage however postmarking data shows benefit

T/F: Studies examining copy number variants (CNV) have related heritability to copies or deletions that affect multiple genes

True Genetic correlation with autism spectrum disorder and overlap with genes involved in schizophrenia and affective disorders

T/F: Dexamethasone has greatest IOP increase; Use the lowest potency, dose, and duration possible

True Lower risk from -Medrysone 1% (hydroxymethylprogesterone) -Loteprednol etabonate 0.2 or 0.5% -Rimexolone 1% -Fluorometholone 0.1%

T/F: Antibodies directed against the Fc portion of Immunoglobulin G (IgG) are also present in other autoimmune disorders, chronic and acute infection, and cancer.

True Normal range: <24 IU/mL Low positive: 24-72 IU/mL (1-3X ULN) High positive: >72 IU/mL (>3X ULN)

T/F: General Rule: All vaccines can be administered at the same visit as all other vaccines

True PPD test must wait 4-6 weeks after live vaccine or give before 2 or more live: if not together must separate 4wks

Uric acid is a waste product of purine metabolism that accumulates if production exceeds excretion. A. True B. False

True PROBLEM: HUMANS DO NOT EXPRESS URICASE

T/F: Only 30-60% of people tolerate Systemic CAIs due to Renal stones, increased uric acid, blood dyscrasias, diuresis, myopia

True Use caution - pulmonary disorders, renal calculi, electrolyte imbalance, renal disease, hepatic disease, diabetes, Addison's disease, sickle-cell disease, high dose salicylates

T/F: If Schedule II for the treatment of pain related to a traumatic injury with a severity score of 9 or greater, the practitioner must prescribe an emergency opioid antagonist along with the controlled substance.

True opioids lead to hyperalgesia

_____________ is also available in suppository form - great option for patients who are vomiting, having a febrile seizure, or will not take oral medications

Tylenol

NON-ROUTINE VACCINES include what?

Typhoid - Vivotif (oral, live); Typhim Vi (inactivated, injection) Tuberculosis Bacille Calmette-Guerin (BCG) - Live; not in US generally Cholera - Vaxchora; live; oral; stored in freezer; reconstitute within 15 minutes

______ is generally given with a photosensitizer (PO methyoxypsoralen - PUVA) to enhance efficacy.

UVA

Which of the following medications have BBW as such below? -Serious infections -Malignancies -Thrombosis

Upadacitinib (Rinvoq) Baricitinib (Olumiant)

NSAIDs: Adverse Effects (systemic)

Upper GI bleeding ▪Counsel patients to watch for black tarry stools ▪Less with COX-2 inhibitors Acute renal failure (Monitor SrCr; hydrate!!!) Cardiac risks ▪Heart attack, stroke, fluid retention, hypertension.... ▪Previously thought to be the worst with COX-2 inhibitors - latest research shows all NSAID increase risks Tinnitus - with salicylates primarily Topical NSAIDs theoretically reduce risk of systemic side effects

When would it be appropriate to treat asymptomatic hyperuricemia?

Uric acid level >10 mg/dL

Treatment for Liver disease in CF:

Ursodeoxycholic acid (Ursodiol)

NAS Use of Treatment Protocols:

Use of a standardized protocol may be more important than drug choice What does the data show? -Shorter duration of treatment -Shorter length of stay -Fewer infants were receiving medication at time of discharge

Anticitrullinated protein antibody (ACPA) may be present before symptoms of RA however is another antibody that's produced in most RA patients, present during inflammation, as well as what?

Used as a diagnostic aid, not definitive Positive antibodies = poorer prognosis Serology -Normal value - < 20 EU/mL -20-39 EU/mL - Weakly positive -40-59 EU/mL - Moderately positive -> 60 EU/mL - Strongly positive

Topicals: Capsaicin Works by decreasing substance P, glutamate (Patches, creams, gels, liquids, lotions) What is it used for?

Used for muscle/joint pain, and neuropathic pain -Many OTC concentrations (0.023-0.25%) ▪Generally applied 3-4 times daily ▪Requires regular treatment x2-4 weeks - not PRN! -Burns! ▪Should go away after regular use ▪Start with lower concentrations

Antidepressants

Used for neuropathic pain or other types of chronic pain, even in absence of depression -Analgesic effect trial = 1 week minimum -Taper off - do not abruptly discontinue TCAs -- Good evidence, limiting side effects ▪Nortriptyline (less SEs?) ▪Amitriptyline SNRIs -- Good evidence, better side effects ▪Duloxetine ▪Venlafaxine

This drug is administered SQ at weeks 0 and 4, then every 12 weeks as maintenance and have the following dose changes below: -Adults <100kg: 45mg SQ -Adults >100kg: 90mg SQ

Ustekinumab (Stelara)

AE of Ipratropium in the treatment of asthma exacerbations: **DuoNeb = Albuterol + ipratropium combination product

Usually minimal systemic effects Given in addition to albuterol and systemic steroids upon initial presentation to ED no clear benefit to continuing ipratropium in hospital

General prophylaxis:

Vaccinate!! 7 day dosing (see initial dosing table) -Though, recommendation is for each day during exposure and 7 days AFTER last exposure -Do not start if >48 hours has passed since initial exposure to influenza -If fever or respiratory illness starts, must seek medical evaluation

This vaccine is Live, SQ and MUST be used within 30min. of reconstituting as well as kept frozen and out of light:

Varicella Vaccine 1st dose: 12-18mos; 2nd dose: 4-6yrs; Rec.: ppl in close contact w/ immunocompromise

The following characteristics below refer to which influenza stage? -Adults: 1 day before until 7 days after illness onset -Children: 10 or more days -Severely immunocompromised: weeks or months

Viral shedding

Viral vaccines - two types:

Virus itself in weakened form. Ex: are Measles and polio Virus itself in inactivated form - it is uninfectious using chemicals, such as formaldehyde, or heat Ex: hepatitis A and B vaccines

When should NSAIDs be initiated for acute gout attacks?

Within 24hrs of onset continue until complete resolution then taper* resolution of symptoms occurs in 5-8 days*

Prescribers must register with DEA and have a DEA # starting with _____ to prescribe Buprenorphine/naloxone (Suboxone, Bunavail, Zubsolv)

X Used for opioid-addiction treatment (alt. to methadone)

COVID-19 mRNA vaccines give instructions for our cells to make _______________ of what is called the "spike protein." The spike protein is found on the surface of the virus that causes COVID-19.

a harmless piece four main sub-groupings of coronaviruses, known as alpha, beta, gamma, and delta.

5% of patients discontinue __________ due to AEs.

allopurinol

Corneal Verticillata is a Clockwise, whorl-like pattern of golden brown or grey deposits that Bind to cellular lipids of the basal epithelial layer of the cornea because of cationic, amphiphilic nature. This is often caused by?

amiodarone, indomethacin, hydroxycholoroquine Found in 20% of netarsudil patients Also called vortex keratopathy

Pharmacologic Treatments for treating lupus (like most other autoimmune conditions) has long been an art more than a science. The only FDA approved treatments are?

aspirin, prednisone, hydroxychloroquine, and belimumab Even other "standard of care" agents are used off-label Optimal doses and durations for induction and maintenance of response have yet to be determined

American College of Rheumatology White Paper A "_____________" is a biologic product that is extremely similar to an approved reference biologic and doesn't not have clinically significant differences in efficacy, adverse effects, or immunogenicity compared to the reference product

biosimilar -An approved biosimilar should produce clinical outcomes comparable to those achieved with its reference product -Providers are encouraged to integrate biosimilars into their practices with the goal of quality health care at lower cost to patients

Know Cutaneous manifestations main symptoms include what?

butterfly rash, photosensitivity, discoid rash, Reynaud's phenomenon Other symptoms include vasculitis, livedo reticularis, periungual erythema, alopecia

Blood Brain Barrier Permeability Neonatal brains thought to display higher permeability due to what?

changes in blood flow and pore density Higher passive permeability of caffeine, cefotaxime, ceftriaxone, acyclovir

Antibodies against blood cells cause various _______________.

cytopenias

Rotavirus is one of the leading causes of ____________ in the United States and the entire world. Severe, prolonged cases (duration 3-9 days) often lead to dehydration and possibly death.

diarrhea RotaTeq and Rotarix are both live vaccines that are administered orally. Almost all children will be infected at least once (and many will have multiple cases) during their first 5 years of life. While the incidence in the United States is considerable, the worldwide rates are significantly higher.

Good results observed with _____________ in refractory lupus arthritis

etanercept (Enbrel) Abatacept (Orencia) - inhibits T-cell co-stimulation - has differing results in various trials

FDA strengthened warning regarding non-aspirin NSAIDs increasing the chance of a what?

heart attack or stroke

Many RA patients form antibodies to self called rheumatoid factors but this does not correlate with level of disease activity however it is used to do what?

help make a diagnosis Seropositive patients tend to have a more aggressive disease

Volume of Distribution Hydrophilic medications (aminoglycosides, linezolid) have ___________ apparent Vd in neonates and infants than in children

higher Gentamicin weight based doses are comparable to adults due to higher Vd in neonates and infants

Protein Binding •Reduced amount of albumin and α1-glycoprotein •Reduced binding affinity •Competition of protein binding sites with bilirubin (sulfamethoxazole and ceftriaxone) These factors lead to what?

higher free fraction of drug that may be clinically significant for highly protein bound drugs (phenytoin) Physiological differences in protein binding are apparent in neonates and infants under 6 months of age.

In OA joints, Synovial fluid is more abundant, less viscous, and the concentration of ______________ is diminished

hyaluronic acid

Counseling points for Corticosteroids are what?

increased risk of infection, HTN, hyperglycemia, weight gain, bone loss, psychiatric concerns, fluid retention

Dysregulation of TNF-α production is associated with _____________. E.g. RA, IBD, ankylosing spondylitis, PsA, and psoriasis

inflammation •Etanercept (Enbrel) •Infliximab (Remicade) •Adalimumab (Humira) •Certolizumab pegol (Cimizia) •Golimumab (Simponi)

ceftriaxone and sulfamethazole must be used with caution in neonates and infants under 3 months of age since it can displace bilirubin from albumin binding sites and increase the neonates risk for _________________.

kernicterus causes a specific form of bilirubin induced encephalopathy and can lead to permanent brain damage. The neonates permeable BBB increases the risk for kernicterus and this leads us to our 3rd factor affecting drug distribution.

Colchicine is *contraindicated* in renal OR hepatic impairment PLUS use of PgP OR strong CYP3A4 inhibitor due to possible what?

life-threatening or fatal colchicine toxicity.

*KNOW* Biologics/BRMs cannot be used with what?

live or live-attenuated vaccines

Patients of Western or Southern European backgrounds respond as well to:

low-dose cyclophosphamide as high-dose regimens

SLICC classification criteria developed in 2012 to identify SLE patients for clinical studies. Not intended to establish a diagnosis in an individual patient but what?

may help assess likelihood of SLE

Patients with psoriasis are at an increased risk of developing _____________.

metabolic syndrome

Abatacept (Orencia) binds CD80/CD86 receptors on antigen-presenting cells and Inhibits interaction between APC and T-cells, prevents T-cell activation What is this medication indicated for?

moderate to severe RA as monotherapy or combo typically in patients who fail to achieve an adequate response from other DMARDs AEs: headache, nasopharyngitis, dizziness, cough, back pain, hypertension, dyspepsia, urinary tract infection, rash, and extremity pain -Infusion reactions -Increase risk of infection -No live vaccines during and for 3 months after the completion of therapy

Duloxetine (Cymbalta®) SSRI and norepinephrine reuptake inhibitor has evidence for modest improvement in knee OA. This medication is approved for treatment of what?

musculoskeletal pain, may be preferred in patients who have neuropathic pain Other centrally acting agents may have benefit, but evidence is lacking ◦Pregabalin, gabapentin, SSRIs, SNRIs, TCAs

Multiple Sclerosis is an autoimmune disease in which the immune system sees the CNS as foreign. This enables the immune system to target what and do what?

myelin and oligodendrocytes (which create myelin) Renders axons susceptible to damage; if severed, damage is irreversible Distortion and disruption of impulses going to and from the brain Much slower neurotransmission

Fingolimod (Gilenya) is the first ______DMT for MS.

oral 0.5mg po daily

Diclofenac 1% gel for topical use Topical NSAID (diclofenac) Voltaren is not for immediate relief and may take 7 days to work. What is this medication indicated for?

osteoarthritis of hand, wrist, elbow, knee, ankle, or foot -Be aware of the safety concerns of NSAID because of systemic absorption even though topical use is safer -Switch from Rx to OTC in 2020

Aroma oils - do not place in nasal passages (vick's vapor rub) - baby's also breathe out of their nose... If a kid aspirates this, they can get a really bad _____________.

pneumanitis

Neuropathic Pain is caused by impaired processing due to nerve damage and is intermittent or constant; usually chronic. What are the associated symptoms?

shooting, burning, tingling, numbness, sensation of bugs crawling

ER products recently made available as what for Hydrocodone?

single entity (recently joined as C2) Considered a less potent opioid (Starting opioid)

Hydrophilic medications like gentamicin (aminoglycoside) are administered to infants in larger mg/kg doses due to a larger apparent Vd than in older children and adults. As body fat increases and total body water decreases, What happens?

the mg/kg dose of aminoglycosides is reduced substantially as to not cause high plasma concentrations which could lead to renal and ototoxicity. most sensitive to changes in total body water in neonates and infants.

The most common type of psoriasis generally presents as what?

thickened, dry, itchy skin, sharply demarcated with erythematous and/or silver or white scales.

Antiphospholipid antibodies lead to what?

thrombosis and fetal death

Healthy diet is recommended with avoidance of sugar and artificial sweeteners. Promote balance of protein, fresh produce, and fiber. Supplement with what?

zinc and iron °Replacement for deficits °May improve the effectiveness of stimulants

Sexual dysfunction is a common problem for MS pts what are meds. involved for treatment?

¤Sildenafil (Viagra) -- being studied for female pts ¤Tadalafil (Cialis) ¤Cardenafil (Levitra) ¤Alprostadil (Caverject) ¤Intraurethral suppositories (MUSE)

What are Tertiary Symptoms of MS:

¨Financial problems ¨Personal/social problems ¨Vocational problems ¨Emotional problems

Symptomatic Management of MS:

¨Gait difficulties and spasticity ¨Tremor ¨Bowel and bladder symptoms ¨Major depression ¨Sensory symptoms ¨Sexual dysfunction ¨Fatigue ¨Cognition ¨Pseudobulbar palsy

Triggers for MS include:

¨Infections ¨Heat (including fever): Uthoff's phenomenon ¨Sleep deprivation ¨Stress ¨Malnutrition ¨Anemia ¨Concurrent organ dysfunction ¨Exertion ¨Childbirth

What are Secondary Symptoms of MS:

¨Recurrent UTIs ¨Urinary calculi ¨Decubiti and osteomyelitis ¨Osteoporosis ¨Respiratory infections ¨Poor nutrition ¨Depression

Daytrana: Methylphenidate patch

°Worn for 9 hours for 12 hour effect °Remove at least three hours before bedtime °Apply to hip area and change the site each day °May cause leukoderma - skin lightening -Quillavant XR ER susp. Must be reconstituted by pharm -Quillivant ER - chewable tablets -Contempla XR-ODT - long acting ODT

Facts about Lupus include:

•1.5 million Americans; 5 million worldwide have a form of lupus •9 out of 10 adults with lupus are women •1 in 3 lupus patients suffer from multiple autoimmune diseases •2-3 times more prevalent among women of color •10-15% of people with lupus will die prematurely •$50,000 lost annually by each lupus patient in healthcare costs and lost productivity

Treatment goals of RA

•Achieve remission or lower disease activity (Treat to Target) •Reduce progression of disease •Reduce risk of joint damage & maintain function of joints •QOL-pain

Treatment for Hematologic manifestations

•Acute: Steroids, IVIG •Maintenance: Mycophenolate, Azathioprine, Cyclosporine •Refractory: Rituximab, Cyclophosphamide

Non-Pharmacologic Treatment Options ALLERGIES

•Allergen avoidance •HEPA Filters Removes pollen, mold spores and cat allergens •Neti Pot May relieve nasal stuffiness, rhinorrhea or sneezing Nei Pot Cautions: be sure to use sterile water; Dry out the pot completely ...Or you can give yourself your own bacterial infection

Uncomplicated Urinary Tract Infections ADDITIONAL OPTIONS:

•Amoxicillin: narrow agent, depends on local susceptibility •Amoxicillin/clavulanate: broad spectrum, consider susceptibility •Nitrofurantoin: very good susceptibility, ESBL, NOT for pyelonephritis •Ciprofloxacin: broad spectrum, reserve for pseudomonas or MDR gram-negative organism or severe penicillin allergy SMX/TMP: depends on local susceptibility If cephalexin is not an option due to accessibility or compliance: •Cepfrozil: good drug level in urine, twice daily, cost effective but verify retail pharmacy stocks it •Cefuroxime: good drug level in urine, twice daily, suspension no longer available Cefazolin susceptibility is a surrogate to predict the efficacy of all oral cephalosporins (e.g., cephalexin, cefprozil, etc.) for the treatment of UTIs due to E. coli, K. pneumoniae, and P. mirabilis**

Vaccine changes due to:

•Antigenic Drift - annual changes •Antigenic Shift - drastic changes = pandemic Reason for new vaccine every year

PATIENT CASE - SHELLY (did not get flu shot this yr.) 55 year old female CC: Cough, aches, feels hot but then cold Symptoms started yesterday PMH: HTN, Type 2 diabetes Meds: lisinopril 20 mg daily metformin ER 1000 mg daily Women's MVI daily

•Based on her symptoms offer her a rapid influenza and COVID test •At home test just approved- takes 2-3 days •No testing available for colds •Offer her influenza vaccine - IIV •She is a candidate for treatment (flu test +, COVID19 (-) and symptoms <48 hours •Higher risk due to HTN and DM2 •She does not want inhaler and wants "one and done" dosing so go with Xofluza oral tablets

POST-VACCINE SAFETY

•Be prepared for acute reaction -Syncope -Acute anaphylaxis •Be sure your CPR is up to date •Have epinephrine on hand and check expiration dates •Observe patient for at least 15 minutes post-vaccine

Uncomplicated CAP ALTERNATIVES OUTPATIENT: NOT RECOMMENDED

•Clindamycin: CAUTION==> resistance •Macrolides: NOT recommended (high resistance) •Doxycycline and TMP/SMX(BACTRIM): NOT recommended (< 50% activity against S.pneumoniae)

NAS Finnegan Score

•Evaluate the neonate q3h •Includes signs and symptoms present at the time of the evaluation •Initiate pharmacologic therapy if 2 - 3 scores > 8 or one score > 12 •Once controlled, scores are used to wean therapy

Non-Pharmacologic Treatment Options COUGH & COLD Increase fluid intake, rest, nutritious diet, increased humidification as well as what could be beneficial?

•Food products: tea with lemon & honey, chicken soup, hot broths •Aroma oils: ease nasal congestion -Camphor, menthol, or eucalyptus •Using a bulb syringe to suction nasal passages -Use saline drops first Honey - cannot use in infants (<1 y.o.) due to concern for infant botulism

What DMARDs are limited by toxicities and LT benefits?

•Gold salts •Azathioprine •D-penicillamine •Cyclosporine •Minocycline •Cyclophosphamide

INFLUENZA- PREVENTION

•Hand hygiene - soap/water or alcohol sanitizer •Avoid touching eyes, nose, mouth •Avoiding others when sick •Sneeze into tissue or shirt sleeve then disinfect hands •Face masks YEARLY VACCINE!

Prognostic (risk) factors of RA:

•High tender and swollen joint counts •Radiographic erosions •Elevated RF / ESR / CRP •Elevated anti-CCP antibodies •Age •Female •Genetics •Tobacco use •Worsening physical function

Presumed Influenza CAP

•Influenza antiviral therapy administered as soon as possible to children with moderate to severe CAP consistent with influenza during widespread local circulation of influenza viruses •Oseltamivir preferred

Uncomplicated CAP ALTERNATIVES OUTPATIENT: FOR SEVERE PENICILLIN ALLERGY AND RESISTANT S. PNEUMONIAE:

•Levofloxacin •Linezolid

Self-Care Options TEETHING Goal is to relieve gum pain and irritation. Non-Pharm options include what?

•Massage the gums •Cold teething ring: Do NOT boil •Dry toast or teething biscuits Teething is a normal physiologic process

Clinical presentation for RDS (respiratory distress syndrome) If given a patient case, be able to identify if the patient is at high risk or not and therefore be able to identify appropriate treatment Risk factors include?

•Prematurity •Male gender •Maternal diabetes •Perinatal asphyxia

Inpatient Empiric Therapy complicated CAP Empiric treatment:

•Similar to that for CAP without effusion •Ampicillin or Ceftriaxone + Clindamycin or SMX/TMP or Vancomycin •Vancomycin has less reliable lung penetration but can be considered if confirmed MRSA bacteremia, colonized or history of MRSA or patient is hemodynamically unstable

Clinical presentation for RDS (respiratory distress syndrome) If given a patient case, be able to identify if the patient is at high risk or not and therefore be able to identify appropriate treatment Signs and symptoms include?

•Tachypnea •Grunting •Nasal flaring •Intercostal retractions •Hypoxia •Hypercarbia •Cyanosis •Opaque infiltrates

PREVENTING SHOULDER INJURY RELATED TO VACCINE ADMINISTRATION (SIRVA) WITH IM INJECTIONS

•Target the central portion of deltoid •Avoid the top 1/3 of the deltoid

Treatment for Skin manifestations:

•Topical or oral steroids •Topical calcineurin inhibitors •Hydroxychloroquine •Methotrexate •Retinoids •Dapsone •Mycophenolate

Fibromyalgia consists of a widespread musculoskeletal pain, fatigue, insomnia, +/- cognitive symptoms. Treatment options associated with this condition include (Patient education, Exercise, Medications). Therapeutic options with insomnia complaints what?

▪Amitriptyline 10mg PO QHS, titrate to 25-50mg/day ▪Pregabalin (Lyrica) 25-50mg PO QHS, up to 300-450mg/day

Fibromyalgia consists of a widespread musculoskeletal pain, fatigue, insomnia, +/- cognitive symptoms. Treatment options associated with this condition include (Patient education, Exercise, Medications). Therapeutic options with fatigue complaints what?

▪Duloxetine (Cymbalta) 30mg PO QAM ==> 60 mg daily ▪Milnacipran (Savella) 12.5mg PO QAM ==> 50mg PO BID -Avoid use with IV digoxin!

This medication is preferred in renal impairment, has many routes but not PO, contains a transdermal patch that is not for acute or opioid naive patients and has what rapid acting formulations for cancer pain in non opioid naive paitents?

▪Sublingual (Abstral, Subsys) ▪Intranasal (Lazanda) ▪Bucal (Fentora, Onsolis, Actiq)

LIVE VACCINE KNOWLEDGE REVIEW

► frozen ►Avoid in pregnancy and immunosuppressed patients ►Can be given together; however, if not at same time, must separate by 4 weeks ►Generally not administered until 12 months of age ►Less doses needed for immunity development ►Oral live vaccines do not always have same warnings as injection live vaccines

HAEMOPHILUS INFLUENZAE TYPE B (HIB)VACCINES ►IM Conjugated Polysaccharide Vaccine ►Routine vaccine in children less than 5 years old What are the vaccine and dose series?

►ActHIB and Hiberix requires 3-dose primary series (2, 4 & 6 months) followed by booster (15-16 months) ►PedvaxHIB requires 2-dose primary series (2 and 4 months) followed by booster (12-15 months) Use in immunocompromised adults ►Asplenic ►Sickle Cell Disease ►Stem Cell Transplant recipients

PERTUSSIS IM (P in Dtap and Tdap) illness cased by bacteria Bordetella pertussis (Whooping cough sound). Who should receive this vaccine?

►All pregnant women for every pregnancy May be used as booster in place of Td every 10 years ►Generally given at 2, 4, and 6 months of age - IM -4th shot at age 15-18 months -5th shot at age 4-6 years ►Additional dose at age 11-18 years is rec. (Tdap) -One additional dose for all adults is recommended Do not worry about amount of time between Td and Tdap

Tdap has lower diphtheria and acellular pertussis dose to avoid local reactions in adults, What are FDA approved age indications?

►DTaP younger than 7 years ►Tdap -Boostrix (GSK) 10 years or older -Adacel (Sanofi Pasteur) 11 to 64 years ACIP Recommendations for individuals 65 years and older (When feasible, use Boostrix)

Diphtheria IM (The D in DT, Td, Dtap and Tdap) protects against illness caused by Corynebacterium diphtheriae. This is normally combined with tetanus, pertussis, and can be combined with HepB. When is this shot given?

►Generally given at 2, 4, and 6 months of age - IM -4th shot at age 15-18 months -5th shot at age 4-6 years ►Booster doses given once every 10 years -Given as Td OR Tdap

What are immunizing concerns with pregnancy population:

►Most vaccines are category C ►Live vaccines generally are avoided ►Inactivated vaccines given when benefit > risk ►Influenza is recommended if pregnant during flu season ►Tdap between weeks 27 and 36 of gestation (every pregnancy) ►Early part of this window preferred ►Post-partum: Rubella and varicella if not previously vaccinated

SUMMARY OF RECOMMENDATION FOR 65 AND OLDER:

►PPSV23 - Recommended for all adults at age 65 -If given recently, must wait 5 years before giving the final dose ►PCV13 - Recommended based on shared clinical decision-making at age 65 -Not needed if received anytime during adulthood -If giving this at age 65, best to do PCV13, wait 12 months, then PPSV23 -If PPSV23 was given first, then wait 12 months before giving PCV13

What are immunizing concerns with immunocompromised population:

►Should NOT receive live vaccines ►Glucocorticoids suppress vaccine response ►Prednisone ≥20mg daily or 2mg/kg daily for at least 2 weeks -Wait 1 month before administering live vaccines ►Other vaccines may be indicated but responses may be suboptimal ►Chronic pulmonary, renal, hepatic, or metabolic disease -Can receive both live and killed vaccines -Often need higher doses & more frequent doses Transplant patients ►When possible, immunize before transplant ►Live vaccines generally not given

Multi-dose Vaccine Series: ►Exception: Do not count doses given before minimum intervals and ages! ►4 days early is allowed

►The minimum interval between most vaccine doses is 4 weeks ►Typically, 5-6 months should elapse between the next-to-the-last and last doses of primary immunizing series ►Increasing the interval between doses does not diminish effectiveness, but does delay the response ►Decreasing the interval between doses may interfere with antibody response and protection ►Delays in immunization do not necessitate restarting series or adding doses

MENINGOCOCCAL B VACCINES IM choice depends on risk of exposure and pts susceptibility to MEN B disease. What are the vaccine and dose series?

►Trumenba (Pfizer) -3-dose series (0, 1-2, 6 months) -2-dose series (0, 6 months) ►Choice depends upon risk of exposure and patient's susceptibility to Men B disease ►Bexsero (Novartis) ►2-dose series (0, 1-6 months)

Warnings/Precautions of Topical Diclofenac Safety:

◦Cardiovascular thrombotic events ◦GI bleeding, ulceration, perforation ◦Hepatotoxicity ◦Hypertension, Heart failure, and edema ◦Renal toxicity and hyperkalemia ◦Serious skin reaction ◦Premature closure of the fetal ductus arteriosus - avoid use after 29 weeks of pregnancy ◦Anemia - increased risk of bleeding with anticoagulants, antiplatelet agents, SSRIs, SNRIs

Contraindications of Topical Diclofenac Safety:

◦Hypersensitivity to diclofenac or other ingredients ◦History of asthma, urticaria, other allergic-type reactions from aspirin or other NSAIDs ◦In the setting of coronary artery bypass graft (CABG) surgery (for 14 days after)

Surgery is known as Arthroplasty (joint replacement) and is for severe OA that has failed responses to meds as well as progressive limitation in ADLs. When is surgery indicated?

◦Radiographic evidence of joint damage ◦Moderate to severe persistent pain or disability ◦Not substantially relieved by an extended course of therapy

What are instructions for using Diclofenac 1% gel for topical use

◦Use hands to gently rub the gel into the skin ◦Hold the dosing card w/ fingertips, rinse, and dry after use ◦Wait 10min. before wearing clothes where applied ◦If treating OA on hands, wait 1hour before washing hands ◦Avoid showering and bathing for 1 hour after application ◦Avoid contact w/ open wounds, eyes, mucous membranes ◦Do not expose to sunlight, tanning, external heat ◦Do not use occlusive dressing ◦Avoid other topical products in same location ◦There is a risk of adverse effects if used with other NSAIDs


Ensembles d'études connexes

Microeconomics Final (multiple choice)

View Set

Chap 6 - Relationship Development and Therapeutic Comm

View Set

English 2 - Test: Passage Response -

View Set